Behavioural Issues

Download as doc, pdf, or txt
Download as doc, pdf, or txt
You are on page 1of 64

PART 3D

BEHAVIOURAL ISSUES
194 QUESTIONS
[1] Source: Publisher
In a responsibility accounting system, the
process in which a supervisor and a
subordinate jointly determine the
subordinate's goals and plans for achieving
these goals is
A. Top-down budgeting.
B. Bottom-up budgeting.
C. Management by objectives.
D. Management by exception.
[2] Source: CMA 1292 3-30
Richmond Enterprises is reviewing its
policies and procedures in an effort to
enhance goal congruence throughout the
organization. The processes that are most
likely to encourage this behavior are
A. Participatory budgeting, reciprocal
cost allocation, and
management-by-objective performance
evaluation.
B. Reciprocal cost allocation,
zero-base budgeting, and standard
costing.
C. Cost-based transfer pricing, imposed
budgeting, and activity-based costing.
D. Cost-based transfer pricing,
management-by-objective performance
evaluation, and participatory budgeting.
[3] Source: CMA 0691 3-24
Most firms use return on investment (ROI)
to evaluate the performance of investment
center managers. If top management wishes
division managers to use all assets without
regard to financing, the denominator in the
ROI calculation will be
A. Total assets available.
B. Total assets employed.
C. Working capital plus other assets.
D. Shareholders' equity.
[4] Source: CMA 0691 3-29
The selection of the denominator in the
return on investment (ROI) formula is
critical to the measure's effectiveness.
Which denominator is criticized because it
combines the effects of operating decisions
made at one level of the organization with
financing decisions made at another
organizational level?
A. Total assets employed.

B. Working capital.
C. Total assets available.
D. Shareholders' equity.
[5] Source: CMA 1291 3-9
A segment of an organization is referred to
as an investment center if it has
A. Authority to make decisions affecting
the major determinants of profit
including the power to choose its
markets and sources of supply.
B. Authority to make decisions affecting
the major determinants of profit
including the power to choose its
markets and sources of supply and
significant control over the amount of
invested capital.
C. Authority to make decisions over the
most significant costs of operations
including the power to choose the
sources of supply.
D. Authority to provide specialized
support to other units within the
organization.
[6] Source: CMA 1292 3-24
Managerial performance can be measured in
many different ways, including return on
investment (ROI) and residual income (RI).
A good reason for using RI instead of ROI is
that
A. RI can be computed without regard
to identifying an investment base.
B. Goal congruence is more likely to be
promoted by using RI.
C. RI is well understood and often used
in the financial press.
D. ROI does not take into consideration
both the investment turnover ratio and
return-on-sales percentage.
[7] Source: CMA 0694 3-27
The Stonebrook Company uses a
performance reporting system that reflects
the company's decentralization of decision
making. The departmental performance
reports show actual costs incurred during
the period against budgeted costs. Any
variances from the budget are assigned to
the individual department manager who
controls the costs. Stonebrook is using a
type of system called
A. Transfer-pricing accounting.
B. Flexible budgeting.
C. Responsibility accounting.
D. Activity-based budgeting.
[8] Source: CMA 0694 3-28
DigitalTech uses an accounting system that

charges costs to the manager who has the


authority to make decisions incurring the
costs. For example, if a sales manager
authorizes a rush order that results in
additional manufacturing costs, these
additional costs are charged to the sales
manager. This type of accounting system is
known as
A. Responsibility accounting.
B. Functional accounting.
C. Transfer-pricing accounting.

C. Increasing sales volume while


holding fixed expenses constant.
D. None of the answers is correct.
[13] Source: CMA 0684 4-9
Return on investment (ROI) is a term often
used to express income earned on capital
invested in a business unit. A company's
ROI will increase if
A. Sales increase by the same dollar
amount as expenses and total assets
increase.

D. Contribution accounting.
[9] Source: CMA 1294 3-22
If a manufacturing company uses
responsibility accounting, which one of the
following items is least likely to appear in a
performance report for a manager of an
assembly line?

B. Sales remain the same and expenses


are reduced by the same dollar amount
that total assets increase.
C. Sales decrease by the same dollar
amount that expenses increase.
D. Sales and expenses increase by the
same percentage that total assets
increase.

A. Supervisory salaries.
B. Materials.
C. Repairs and maintenance.
D. Equipment depreciation.
[10] Source: Publisher
In evaluating an investment center, top
management should concentrate on
A. Dollar sales.
B. Net income.

[14] Source: Publisher


To properly motivate divisional
management, the divisional ROIs should be
A. Equal.
B. Greater in the less profitable
divisions to motivate those divisions to
achieve higher ROIs.
C. Lower in more profitable divisions
in which motivation is unnecessary.
D. Different based upon strategic goals
of the firm.

C. Profit percentages.
D. Return on investment.
[11] Source: Publisher
The return on investment calculation
considers only the following components:
S = Sales
I = Investment
NI = Net income
Which of the following formulas best
describes the return on investment
calculation?

[15] Source: Publisher


Goal congruence is
A. The desire and the commitment to
achieve a specific goal.
B. The sharing of goals by supervisors
and subordinates.
C. The extent to which individuals have
the authority to make decisions.
D. The extent of the attempt to
accomplish a specific goal.

A. (I S) x (S NI) = I NI
B. (S I) x (NI S) = NI I
C. (I S) x (NI S) = (I x NI) x (S x S)

[16] Source: Publisher


The proposed transfer price is based upon
the outlay cost. Outlay cost plus opportunity
cost is

D. (S I) x (S NI) = (S x S) (I x NI)
A. The retail price.
[12] Source: Publisher
Which of the following will not improve
return on investment if other factors are
constant?
A. Decreasing expenses or assets.
B. Increasing selling prices.

B. The price representing the cash


outflows of the supplying division plus
the contribution to the supplying
division from an outside sale.
C. The price usually set by an
absorption-costing calculation.

D. The price set by charging for


variable costs plus a lump sum or an
additional markup, but less than full
markup.
[17] Source: Publisher
The proposed transfer price is a cost-plus
price. Variable-cost-plus price is

producing units for B. Division A cannot


increase its sales to outsiders. From the
perspective of the company as a whole,
from whom should Division B acquire the
units, assuming B's market is unaffected?
A. Outside vendors.
B. Division A, but only at the variable
cost per unit.

A. The price on the open market.


B. The price representing the cash
outflows of the supplying division plus
the contribution to the supplying
division from an outside sale.
C. The price set by charging for
variable costs plus a lump sum or an
additional markup, but less than full
markup.
D. The price otherwise paid by an
outsider, recorded by the selling
division but not the buying division.
[18] Source: Publisher
Full-cost price is
A. The price on the open market.
B. The price representing the cash
outflows of the supplying division plus
the contribution to the supplying
division from an outside sale.
C. The price usually set by an
absorption-costing calculation.

C. Division A, but only until fixed costs


are covered, then from outside vendors.
D. Division A, despite the increased
transfer price.
[21] Source: CIA 0592 IV-19
Division Z of a company produces a
component that it currently sells to outside
customers for $20 per unit. At its current
level of production, which is 60% of
capacity, Division Z's fixed cost of
producing this component is $5 per unit and
its variable cost is $12 per unit. Division Y
of the same company would like to purchase
this component from Division Z for $10.
Division Z has enough excess capacity to
fill Division Y's requirements. The
managers of both divisions are compensated
based upon reported profits. Which of the
following transfer prices will maximize
total company profits and be most equitable
to the managers of Division Y and Division
Z?
A. $12 per unit.
B. $18 per unit.

D. The price set by charging for


variable costs plus a lump sum or an
additional markup, but less than full
markup.
[19] Source: CIA 1190 IV-20
A limitation of transfer prices based on
actual cost is that they
A. Charge inefficiencies to the
department that is transferring the
goods.
B. Can lead to suboptimal decisions for
the company as a whole.
C. Must be adjusted by some markup.
D. Lack clarity and administrative
convenience.

C. $20 per unit.


D. $22 per unit.
[22] Source: CIA 0589 IV-16
Division A of a company is currently
operating at 50% capacity. It produces a
single product and sells all its production to
outside customers for $13 per unit. Variable
costs are $7 per unit, and fixed costs are $6
per unit at the current production level.
Division B, which currently purchases this
product from an outside supplier for $12 per
unit, would like to purchase the product
from Division A. Division A will operate at
80% capacity to meet outside customers'
and Division B's demand. What is the
minimum price that Division A should
charge Division B for this product?
A. $7.00 per unit.

[20] Source: CIA 1183 IV-5


A company has two divisions, A and B,
each operated as a profit center. A charges
B $35 per unit for each unit transferred to B.
Other data follow:
A's variable cost per unit
$30
A's fixed costs
$10,000
A's annual sales to B
5,000 units
A's sales to outsiders
50,000 units
A is planning to raise its transfer price to
$50 per unit. Division B can purchase units
at $40 each from outsiders, but doing so
would idle A's facilities now committed to

B. $9.60 per unit.


C. $12.00 per unit.
D. $13.00 per unit.
[23] Source: CIA 0588 IV-19
The alpha division of a company, which is
operating at capacity, produces and sells
1,000 units of a certain electronic
component in a perfectly competitive

market. Revenue and cost data are as


follows:
Sales
$50,000
Variable costs
34,000
Fixed costs
12,000
The minimum transfer price that should be
charged to the beta division of the same
company for each component is
A. $12

storage and handling are $40. The company


president selects a $220 transfer price. This
is an example of
A. Market-based transfer pricing.
B. Cost-based transfer pricing.
C. Negotiated transfer pricing.
D. Cost plus 20% transfer pricing.

B. $34
C. $46
D. $50
[24] Source: CIA 0593 IV-16
Which of the following is the most
significant disadvantage of a cost-based
transfer price?

[27] Source: CIA 0594 III-40


Which of the following is not true about
international transfer prices for a
multinational firm?
A. Allows firms to attempt to minimize
worldwide taxes.
B. Allows the firm to evaluate each
division.

A. Requires internally developed


information.

C. Provides each division with a


profit-making orientation.

B. Imposes market effects on company


operations.

D. Allows firms to correctly price


products in each country in which it
operates.

C. Requires externally developed


information.
D. May not promote long-term
efficiencies.
[25] Source: CIA 1193 IV-18
One department of an organization, Final
Assembly, is purchasing subcomponents
from another department, Materials
Fabrication. The price that will be charged
to Final Assembly by Materials Fabrication
is to be determined. Outside market prices
for the subcomponents are available. Which
of the following is the most correct
statement regarding a market-based transfer
price?
A. Marginal production cost transfer
prices provide incentives to use
otherwise idle capacity.
B. Market transfer prices provide an
incentive to use otherwise idle
capacity.
C. Overall long term competitiveness is
enhanced with a market-based transfer
price.
D. Corporate politics is more of a
factor in a market-based transfer price
than with other methods.
[26] Source: CIA 1193 IV-19
The Eastern division sells goods internally
to the Western division of the same
company. The quoted external price in
industry publications from a supplier near
Eastern is $200 per ton plus transportation.
It costs $20 per ton to transport the goods to
Western. Eastern's actual market cost per
ton to buy the direct materials to make the
transferred product is $100. Actual per ton
direct labor is $50. Other actual costs of

[28] Source: CIA 0594 III-71


Which of the following is not true of
responsibility accounting?
A. Managers should only be held
accountable for factors over which they
have significant influence.
B. The focus of cost center managers
will normally be more narrow than that
of profit center managers.
C. Every factor that affects a firm's
financial performance ultimately is
controllable by someone, even if that
someone is the person at the top of the
firm.
D. When a responsibility account
system exists, operations of the business
are organized into separate areas
controlled by individual managers.
[29] Source: CIA 1191 IV-18
A company plans to implement a bonus plan
based on segment performance. In addition,
the company plans to convert to a
responsibility accounting system for segment
reporting. The following costs, which have
been included in the segment performance
reports that have been prepared under the
current system, are being reviewed to
determine if they should be included in the
responsibility accounting segment reports:
I. Corporate administrative costs allocated on the basis of
net
segment sales.
II. Personnel costs assigned on the basis of the number of
employees
in each segment.
III. Fixed computer facility costs divided equally among each
segment.

IV. Variable computer operational costs charged to each


segment based
on actual hours used times a predetermined standard
rate; any
variable cost efficiency or inefficiency remains in the
computer
department.
Of these four cost items, the only item that
could logically be included in the segment
performance reports prepared on a
responsibility accounting basis would be the

A. Generally accepted accounting


principles.
B. The Financial Accounting Standards
Board.
C. The American Institute of Certified
Public Accountants.
D. Management.

A. Corporate administrative costs.


B. Personnel costs.
C. Fixed computer facility costs.
D. Variable computer operational costs.
[30] Source: CIA 1192 IV-22
An organization employs a system of
internal reporting that furnishes
departmental managers with revenue and
cost information on only those items that are
subject to their control. Items not subject to
the manager's control are not included in the
performance reports. This method of
accounting is known as
A. Contribution margin reporting.
B. Segment reporting.
C. Absorption cost accounting.
D. Responsibility accounting.
[31] Source: CIA 0589 IV-15
In a responsibility accounting system,
managers are accountable for

[34] Source: CIA 1191 IV-17


The receipt of raw materials used in the
manufacture of products and the shipping of
finished goods to customers is under the
control of the warehouse supervisor. The
warehouse supervisor's time is spent
approximately 60% on receiving activities
and 40% on shipping activities. Separate
staffs are employed for the receiving and
shipping operations. The labor-related costs
for the warehousing function are as follows:
Warehouse supervisor's salary
$ 40,000
Receiving clerks' wages
75,000
Shipping clerks' wages
55,000
Employee benefit costs (30% of wage
and salary costs)
51,000
????????
$221,000
========
The company employs a responsibility
accounting system for performance reporting
purposes. The costs are classified on the
report as period or product costs. The total
labor-related costs that would be listed on
the responsibility accounting performance
report as product costs under the control of
the warehouse supervisor for the
warehousing function would be
A. $97,500

A. Variable costs but not for fixed


costs.
B. Product costs but not for period
costs.

B. $128,700
C. $130,000
D. $169,000

C. Incremental costs.
D. Costs over which they have
significant influence.
[32] Source: Publisher
In a responsibility accounting system, a
feedback report that focuses on the
difference between budgeted amounts and
actual amounts is an example of

[35] Source: CIA 1190 IV-21


When comparing the residual income of
several investment centers, the validity of
comparisons may be destroyed by
A. Peculiarities of each investment
center.
B. Consistent use of an imputed interest
rate.

A. Management by exception.
B. Assessing blame.
C. Granting rewards to successful
managers.
D. Ignoring other variables for which
the budgeted goals were met.
[33] Source: Publisher
The format for internal reports in a
responsibility accounting system is
prescribed by

C. Common amounts of invested capital


for each investment center.
D. None of the answers is correct.
[36] Source: Publisher
Transfer pricing should encourage goal
congruence and managerial effort. In a
decentralized organization, it should also
encourage autonomous decision making.
Managerial effort is

A. The desire and the commitment to


achieve a specific goal.
B. The extent to which individuals have
the authority to make decisions.
C. The extent of the attempt to
accomplish a specific goal.
D. The sharing of goals between units
of an organization.

C. 4
D. 20
[41] Source: Publisher
(Refers to Fact Pattern #1)
For Segment A, ROI is
A. 25%
B. 20.8%

[Fact Pattern #1]


C. 33%
Segment A Segment B Segment C
Segment D

D. 8.3%
????????? ????????? ????????? ??????

????
Net income
$5,000
--- $90,000
Sales
60,000 $750,000 $135,000 1,800,000
Investment
24,000 500,000 45,000
-Net income as % of sales
----Turnover of investment
----ROI
--20%
7.5%
Minimum ROI--dollars
---- $120,000
Minimum ROI--%
20%
6%
--Residual income
--0- $2,250
-[37] Source: Publisher
(Refers to Fact Pattern #1)
For Segment B, net income as a percentage
of sales is
A. 8%

[42] Source: Publisher


(Refers to Fact Pattern #1)
For Segment B, ROI is
A. 6%
B. 25%
C. 20%
D. 7.5%
[43] Source: Publisher
(Refers to Fact Pattern #1)
For Segment A, the minimum dollar ROI is

B. 6.67%
A. $30,000
C. 4%
B. $4,800
D. 10%
C. $120,000
[38] Source: Publisher
(Refers to Fact Pattern #1)
For Segment C, net income as a percentage
of sales is
A. 5%

D. $12,000
[44] Source: Publisher
(Refers to Fact Pattern #1)
For Segment B, the minimum dollar ROI is

B. 6.67%

A. $30,000

C. 4%

B. $6,750

D. 20%

C. $4,800
D. $45,000

[39] Source: Publisher


(Refers to Fact Pattern #1)
For Segment C, the turnover of investment is
A. 3

[45] Source: Publisher


(Refers to Fact Pattern #1)
For Segment C, the minimum dollar ROI is

B. 1.5

A. $6,750

C. 2.5

B. $4,800

D. 4

C. $120,000
D. $9,000

[40] Source: Publisher


(Refers to Fact Pattern #1)
For Segment D, the turnover of investment is
A. 1.5
B. 2.5

[46] Source: Publisher


(Refers to Fact Pattern #1)
Assume that the minimum dollar ROI is
$6,750. In Segment C, the minimum
percentage of ROI is

A. 20%

Cost
Investment Profit
Centers Centers
Centers
??????? ?????????? ???????
A.

B. 6%
C. 15%
D. 10%

Yes

Yes

Yes

Yes

No

No

No

Yes

Yes

No

No

No

B.

[47] Source: Publisher


(Refers to Fact Pattern #1)
In Segment D, the minimum percentage of
ROI is

C.
D.

A. 20%
B. 6%

[52] Source: Publisher


Periodic internal reports used for
performance evaluation purposes and based
on a responsibility accounting system should
not include

C. 15%
D. 10%
[48] Source: Publisher
(Refers to Fact Pattern #1)
In Segment A, the residual income is

A. Allocated fixed overhead.


B. A distinction between controllable
and noncontrollable costs.

A. $200
C. An organization chart.
B. $12,000
D. Variances between actual and
budgeted controllable costs.

C. $(30,000)
D. $(60,000)
[49] Source: Publisher
(Refers to Fact Pattern #1)
In Segment D, the residual income is
A. $12,000.

[53] Source: Publisher


Responsibility centers accumulate costs for
control purposes when using
Process Job-Order
ActivityCosting Costing Based Costing
??????? ????????? ?????????????
A.

B. $(30,000)
C. $(60,000)

Yes

Yes

Yes

Yes

No

Yes

No

Yes

No

No

No

No

B.

D. $9,000
C.
[50] Source: Publisher
Production levels are expected to increase
within a relevant range. A flexible budget is
used. What are the anticipated effects on the
following?
Fixed Costs
per Unit
???????????
A.

Variable Costs
per Unit
??????????????

Increase

Increase

Increase

No change

Decrease

Decrease

Decrease

No change

B.
C.
D.

D.

[54] Source: Publisher


Harris Co.'s income statement for profit
center No. 12 for August includes
Contribution margin
$84,000
Period expenses:
Manager's salary
$24,000
Depreciation on
accommodations
9,600
Allocated corporate expenses 6,000 (39,600)
??????? ????????
Profit center income
$44,400
========
The profit center's manager is most likely
able to control which of the following?
A. $84,000

[51] Source: Publisher


Which of the following types of
responsibility centers include controllable
revenues in their performance reports?

B. $68,400
C. $60,000

D. $44,400
[55] Source: Publisher
Suboptimization occurs when a management
decision benefits the
Company
Profit Investment
As a Whole Center
Center
?????????? ?????? ??????????
A.
Yes

Yes

Yes

Yes

No

No

No

No

No

Direct labor
180,000
Factory overhead
84,000
$ 50,400
General, selling, and
administrative
36,000
57,600
????????
????????
Total
$540,000
$108,000
========
========
During year 2, Ash produced 360,000
hammers, which were sold for $2.40 each.
Georgia's investment in Ash was $600,000
and $840,000 at January 1, year 1 and
January 1, year 2, respectively. Georgia
normally imputes interest on investments at
18% of average invested capital.

B.
C.
D.

[58] Source: Publisher


(Refers to Fact Pattern #2)
For the year ended December 31, year 2,
Ash's return on average investment was
A. 18%

No

Yes

Yes

B. 25.7%
C. 30%

[56] Source: Publisher


The following information pertains to
Andrew Co. for the year ended December
31:
Sales
$720,000
Net income
120,000
Capital investment
480,000
To determine Andrew's return on
investment, which of the following
equations should be used?
A. (720,000/480,000) x
(720,000/120,000) = ROI

D. 36%
[59] Source: Publisher
(Refers to Fact Pattern #2)
Assume that Ash's net operating income was
$72,000 and its average invested capital
was $720,000. Determine Ash's residual
income (loss) for the year ended December
31, year 2.
A. $(57,600)
B. $(79,200)

B. (480,000/720,000) x
(720,000/120,000) = ROI
C. (720,000/480,000) x
(120,000/720,000) = ROI
D. (480,000/720,000) x
(120,000/720,000) = ROI
[57] Source: Publisher
The following information relates to Cinder
Co.'s Northeast Division:

C. $72,000
D. $(36,000)
[60] Source: Publisher
(Refers to Fact Pattern #2)
How many hammers did Ash have to sell in
year 2 to break even?
A. 120,000
B. 110,000

Sales
$600,000
Variable costs
360,000
Traceable fixed costs
60,000
Average invested capital
120,000
Imputed interest rate
8%
Cinder's residual income was
A. $170,400
B. $180,000

C. 100,000
D. 90,000
[61] Source: Publisher
(Refers to Fact Pattern #2)
Ash's contribution margin for the year ended
December 31, year 2 was

C. $189,600

A. $350,000

D. $230,400

B. $324,000
C. $309,600

[Fact Pattern #2]


Georgia Co.'s tool division, Ash, incurred
the following costs for year 2:
Variable
Fixed
????????
????????
Direct materials
$240,000

D. $216,000
[62] Source: Publisher
(Refers to Fact Pattern #2)
Based on Ash's year 2 financial information,

and a projected year 3 production level of


400,000 hammers, what will Ash's
estimated year 3 total costs and expenses be,
assuming variable costs remain at $1.50 per
unit?

[66] Source: CMA 1293 3-17


The budgeting process that uses management
by objectives and input from the individual
manager is an example of the application of

A. $648,000
A. Flexible budgeting.
B. $708,000
B. Human resource management.
C. $718,800
C. Responsibility accounting.
D. $720,000
D. Capital budgeting.
[63] Source: CIA 0594 III-44
A firm prepared a segmented income
statement that included the following data
for its suburban marketing segment:
Fixed costs controllable by the suburban
marketing segment manager
$150,000
Fixed suburban marketing costs
controllable by corporate management $250,000
Fixed manufacturing costs allocated to
the suburban marketing segment
$110,000
Variable manufacturing costs
$200,000
Variable selling costs
$100,000
Variable administrative costs
$130,000
Net sales
$950,000
The best measure of the economic
performance of the suburban marketing
segment is:
A. $370,000

[67] Source: CMA 1293 3-21


A successful responsibility accounting
reporting system is dependent upon
A. The correct allocation of
controllable variable costs.
B. Identification of the management
level at which all costs are
controllable.
C. The proper delegation of
responsibility and authority.
D. A reasonable separation of costs into
their fixed and variable components
since fixed costs are not controllable
and must be eliminated from the
responsibility report.

B. $10,000
C. $520,000
D. $120,000
[64] Source: CMA 1291 3-10
A segment of an organization is referred to
as a service center if it has

[68] Source: CMA 0694 3-24


Decentralized firms can delegate authority
and yet retain control and monitor managers'
performance by structuring the organization
into responsibility centers. Which one of the
following organizational segments is most
like an independent business?
A. Revenue center.

A. Responsibility for developing


markets and selling the output of the
organization.

B. Profit center.

B. Responsibility for combining the raw


materials, direct labor, and other
factors of production into a final output.

D. Investment center.

C. Authority to make decisions affecting


the major determinants of profit
including the power to choose its
markets and sources of supply.
D. Authority to provide specialized
support to other units within the
organization.
[65] Source: CMA 0693 3-14
The least complex segment or area of
responsibility for which costs are allocated
is a(n)

C. Cost center.

[69] Source: CMA 1294 3-20


Fairmount Inc. uses an accounting system
that charges costs to the manager who has
been delegated the authority to make the
decisions incurring the costs. For example,
if the sales manager accepts a rush order
that will result in higher than normal
manufacturing costs, these additional costs
are charged to the sales manager because the
authority to accept or decline the rush order
was given to the sales manager. This type of
accounting system is known as
A. Responsibility accounting.
B. Functional accounting.

A. Profit center.
C. Reciprocal allocation.
B. Investment center.
D. Transfer price accounting.
C. Contribution center.
D. Cost center.

[70] Source: CMA 1294 3-21

Sherman Company uses a performance


reporting system that reflects the company's
decentralization of decision making. The
departmental performance report shows one
line of data for each subordinate who
reports to the group vice president. The data
presented show the actual costs incurred
during the period, the budgeted costs, and
all variances from budget for that
subordinate's department. Sherman is using
a type of system called

line of data for each subordinate who


reports to the group vice president. The data
presented show the actual costs incurred
during the period, the budgeted costs, and
all variances from budget for that
subordinate's department. The WK Company
is using a system called
A. Flexible budgeting.
B. Responsibility accounting.

A. Contribution accounting.

C. Cost-benefit accounting.

B. Cost-benefit accounting.

D. Program budgeting.

C. Flexible budgeting.
D. Responsibility accounting.
[71] Source: CMA 0695 3-28
In responsibility accounting, a center's
performance is measured by controllable
costs. Controllable costs are best described
as including
A. Direct material and direct labor,
only.
B. Only those costs that the manager can
influence in the current time period.
C. Only discretionary costs.
D. Those costs about which the manager
is knowledgeable and informed.
[72] Source: CMA 1295 3-5
Responsibility accounting defines an
operating center that is responsible for
revenue and costs as a(n)
A. Profit center.
B. Revenue center.

[75] Source: CMA 0691 3-28


The basic purpose of a responsibility
accounting system is
A. Budgeting.
B. Motivation.
C. Authority.
D. Variance analysis.
[76] Source: CMA 0693 3-29
Which one of the following firms is likely to
experience dysfunctional motivation on the
part of its managers due to its allocation
methods?
A. To allocate depreciation of forklifts
used by workers at its central
warehouse, Shahlimar Electronics uses
predetermined amounts calculated on
the basis of the long-term average use
of the services provided.
B. Manhattan Electronics uses the sales
revenue of its various divisions to
allocate costs connected with the
upkeep of its headquarters building. It
also uses ROI to evaluate the divisional
performances.

C. Division.
D. Operating unit.
[73] Source: CMA 1296 3-16
Rockford Manufacturing Corporation uses a
responsibility accounting system in its
operations. Which one of the following
items is least likely to appear in a
performance report for a manager of one of
Rockford's assembly lines?
A. Direct labor.

C. Rainier Industrial does not allow its


service departments to pass on their
cost overruns to the production
departments.
D. Tashkent Auto's MIS is operated out
of headquarters and serves its various
divisions. Tashkent's allocation of the
MIS-related costs to its divisions is
limited to costs the divisions will incur
if they were to outsource their MIS
needs.

B. Materials.
C. Repairs and maintenance.
D. Depreciation on the manufacturing
facility.
[74] Source: CMA 0692 3-23
The WK Company uses a performance
reporting system that reflects the company's
decentralization of decision making. The
departmental performance report shows one

[77] Source: CMA 0686 4-14


The segment margin of the Wire Division of
Lerner Corporation should not include
A. Net sales of the Wire Division.
B. Fixed selling expenses of the Wire
Division.
C. Variable selling expenses of the
Wire Division.

D. The Wire Division's fair share of the


salary of Lerner Corporation's
president.

C. Working capital.
D. Shareholders' equity.

[78] Source: CIA 0587 IV-15


Overtime conditions and pay were recently
set by the personnel department. The
production department has just received a
request for a rush order from the sales
department. The production department
protests that additional overtime costs will
be incurred as a result of the order. The
sales department argues that the order is
from an important customer. The production
department processes the order. To control
costs, which department should never be
charged with the overtime costs generated
as a result of the rush order?
A. Personnel department.
B. Production department.
C. Sales department.
D. Shared by production department
and sales department.
[79] Source: CMA 0691 3-26
If a manufacturing company uses
responsibility accounting, which one of the
following items is least likely to appear in a
performance report for a manager of an
assembly line?

[82] Source: CMA 1291 3-7


Which one of the following items would
most likely not be incorporated into the
calculation of a division's investment base
when using the residual income approach
for performance measurement and
evaluation?
A. Fixed assets employed in division
operations.
B. Land being held by the division as a
site for a new plant.
C. Division inventories when division
management exercises control over the
inventory levels.
D. Division accounts payable when
division management exercises control
over the amount of short-term credit
used.
[83] Source: CMA 1292 3-21
A firm earning a profit can increase its
return on investment by
A. Increasing sales revenue and
operating expenses by the same dollar
amount.

A. Supervisory salaries.
B. Materials.

B. Decreasing sales revenues and


operating expenses by the same
percentage.

C. Repairs and maintenance.


D. Depreciation on equipment.
[80] Source: CMA 1292 3-22
When using a contribution margin format for
internal reporting purposes, the major
distinction between segment manager
performance and segment performance is
A. Unallocated fixed cost.
B. Direct variable costs of producing
the product.
C. Direct fixed cost controllable by the
segment manager.
D. Direct fixed cost controllable by
others.
[81] Source: CMA 0691 3-29
The selection of the denominator in the
return on investment (ROI) formula is
critical to the measure's effectiveness.
Which denominator is criticized because it
combines the effects of operating decisions
made at one level of the organization with
financing decisions made at another
organizational level?
A. Total assets available.
B. Total assets employed.

C. Increasing investment and operating


expenses by the same dollar amount.
D. Increasing sales revenues and
operating expenses by the same
percentage.
[Fact Pattern #3]
Edith Carolina, president of the Deed
Corporation, requires a minimum return on
investment of 8% for any project to be
undertaken by her company. The company is
decentralized, and leaves investment
decisions up to the discretion of the division
managers as long as the 8% return is
expected to be realized. Michael Sanders,
manager of the Cosmetics Division, has had
a return on investment of 14% for his
division for the past 3 years and expects the
division to have the same return in the
coming year. Sanders has the opportunity to
invest in a new line of cosmetics which is
expected to have a return on investment of
12%.
[84] Source: CMA 0693 3-12
(Refers to Fact Pattern #3)
If the Deed Corporation evaluates
managerial performance using residual
income based on the corporate minimum
required rate of return, what will be the
preference for taking on the proposed
cosmetics line by Edith Carolina and
Michael Sanders?

Carolina Sanders
???????? ???????
A.
Accept

B. Historical weighted-average cost of


capital for the company.
C. Target return on investment set by the
company's management.

Reject

B.

D. Average return on investments for


the company over the last several years.
Reject

Accept

Accept

Accept

C.
[88] Source: CMA 0694 3-29
D.
Reject

Reject

[85] Source: CMA 0693 3-11


(Refers to Fact Pattern #3)
If the Deed Corporation evaluates
managerial performance using return on
investment, what will be the preference for
taking on the proposed cosmetics line by
Edith Carolina and Michael Sanders?
Carolina
????????
A.

Sanders
???????

Accept

Reject

Reject

Accept

B.
C.
Accept

Accept

Reject

Reject

D.

[86] Source: CMA 0693 3-27


Which one of the following statements
pertaining to the return on investment (ROI)
as a performance measurement is incorrect?

One approach to measuring divisional


performance is return on investment. Return
on investment is expressed as operating
income
A. Divided by the current year's capital
expenditures plus cost of capital.
B. Minus imputed interest charged for
invested capital.
C. Divided by fixed assets.
D. Divided by total assets.
[89] Source: CMA 0695 3-20
REB Service Co. is a computer service
center. For the month of May 1995, REB
had the following operating statistics:
Sales
$450,000
Operating income
25,000
Net profit after taxes
8,000
Total assets
500,000
Shareholders' equity
200,000
Cost of capital
6%
Based on the above information, which one
of the following statements is correct? REB
has a
A. Return on investment of 4%.
B. Residual income of $(5,000).

A. When the average age of assets


differs substantially across segments of
a business, the use of ROI may not be
appropriate.
B. ROI relies on financial measures that
are capable of being independently
verified while other forms of
performance measures are subject to
manipulation.
C. The use of ROI may lead managers
to reject capital investment projects that
can be justified by using discounted
cash flow models.
D. The use of ROI can make it
undesirable for a skillful manager to
take on trouble-shooting assignments
such as those involving turning around
unprofitable divisions.
[87] Source: CMA 0694 3-18
The imputed interest rate used in the
residual income approach to performance
evaluation can best be described as the
A. Average lending rate for the year
being evaluated.

C. Return on investment of 1.6%.


D. Residual income of $(22,000).
[90] Source: CMA 1296 3-2
The segment margin of an investment center
after deducting the imputed interest on the
assets used by the investment center is
known as
A. Return on investment.
B. Residual income.
C. Operating income.
D. Return on assets.
[91] Source: CMA 1296 3-27
Which denominator used in the return on
investment (ROI) formula is criticized
because it combines the effects of operating
decisions made at one organizational level
with financing decisions made at another
organizational level?
A. Total assets employed.

B. Shareholders' equity.
C. Working capital plus other assets.
D. Total assets available.
[92] Source: CMA 0692 3-14
The most fundamental responsibility center
affected by the use of market-based transfer
prices is a(n)
A. Production center.
B. Investment center.
C. Cost center.
D. Profit center.
[93] Source: CMA 0694 3-30
An appropriate transfer price between two
divisions of The Stark Company can be
determined from the following data:
Fabricating Division
Market price of subassembly
$50
Variable cost of subassembly
$20
Excess capacity (in units)
1,000
Assembling Division
Number of units needed
900
What is the natural bargaining range for the
two divisions?

The market price of the video card used by


the Entertainment Division is $10.98 per
unit.
[94] Source: CMA 0696 3-26
(Refers to Fact Pattern #4)
A per-unit transfer price from the Video
Cards Division to the Entertainment
Division at full cost, $9.15, would
A. Allow evaluation of both divisions
on a competitive basis.
B. Satisfy the Video Cards Division's
profit desire by allowing recovery of
opportunity costs.
C. Provide no profit incentive for the
Video Cards Division to control or
reduce costs.
D. Encourage the Entertainment
Division to purchase video cards from
an outside source.
[95] Source: CMA 0696 3-27
(Refers to Fact Pattern #4)
Assume that the Entertainment Division is
able to purchase a large quantity of video
cards from an outside source at $8.70 per
unit. The Video Cards Division, having
excess capacity, agrees to lower its transfer
price to $8.70 per unit. This action would

A. Between $20 and $50.


B. Between $50 and $70.
C. Any amount less than $50.
D. $50 is the only acceptable price.
[Fact Pattern #4]
Parkside Inc. has several divisions that
operate as decentralized profit centers.
Parkside's Entertainment Division
manufactures video arcade equipment using
the products of two of Parkside's other
divisions. The Plastics Division
manufactures plastic components, one type
that is made exclusively for the
Entertainment Division, while other less
complex components are sold to outside
markets. The products of the Video Cards
Division are sold in a competitive market;
however, one video card model is also used
by the Entertainment Division. The actual
costs per unit used by the Entertainment
Division are presented below.
Plastic
Video
Components Cards
?????????? ?????
Direct material
$1.25
$2.40
Direct labor
2.35
3.00
Variable overhead
1.00
1.50
Fixed overhead
.40
2.25
?????
?????
Total cost
$5.00
$9.15
=====
=====
The Plastics Division sells its commercial
products at full cost plus a 25% markup and
believes the proprietary plastic component
made for the Entertainment Division would
sell for $6.25 per unit on the open market.

A. Optimize the profit goals of the


Entertainment Division while
subverting the profit goals of Parkside
Inc.
B. Allow evaluation of both divisions
on the same basis.
C. Subvert the profit goals of the Video
Cards Division while optimizing the
profit goals of the Entertainment
Division.
D. Optimize the overall profit goals of
Parkside Inc.
[96] Source: CMA 0696 3-28
(Refers to Fact Pattern #4)
Assume that the Plastics Division has excess
capacity and it has negotiated a transfer
price of $5.60 per plastic component with
the Entertainment Division. This price will
A. Cause the Plastics Division to
reduce the number of commercial
plastic components it manufactures.
B. Motivate both divisions as estimated
profits are shared.
C. Encourage the Entertainment
Division to seek an outside source for
plastic components.
D. Demotivate the Plastics Division
causing mediocre performance.

[97] Source: CMA 1296 3-17

In theory, the optimal method for


establishing a transfer price is
A. Flexible budget cost.

D. The price set by charging for


variable costs plus a lump sum or an
additional markup, but less than full
markup.

B. Incremental cost.
C. Budgeted cost with or without a
markup.
D. Market price.
[98] Source: CIA 1188 IV-23
The price that one division of a company
charges another division for goods or
services provided is called the
A. Market price.
B. Transfer price.
C. Outlay price.
D. Distress price.
[99] Source: Publisher
Motivation is

[102] Source: CIA 1191 IV-19


A carpet manufacturer maintains a retail
division consisting of stores stocking its
brand and other brands, and a manufacturing
division that makes carpets and pads. An
outside market exists for carpet padding
material in which all padding produced can
be sold. The proper transfer price for
padding transferred from the manufacturing
division to the retail division is
A. Variable manufacturing division
production cost.
B. Variable manufacturing division
production cost plus allocated fixed
factory overhead.
C. Variable manufacturing division
production cost plus variable selling
and administrative cost.
D. The market price at which the retail
division could purchase padding.

A. The desire and the commitment to


achieve a specific goal.
B. The sharing of goals by supervisors
and subordinates.

[103] Source: CIA 1190 IV-20


A limitation of transfer prices based on
actual cost is that they

C. The extent to which individuals have


the authority to make decisions.

A. Charge inefficiencies to the


department that is transferring the
goods.

D. The extent of the attempt to


accomplish a specific goal.

B. Can lead to suboptimal decisions for


the company as a whole.
C. Must be adjusted by some markup.

[100] Source: Publisher


Outlay cost plus opportunity cost is

D. Lack clarity and administrative


convenience.

A. The retail price.


B. The price representing the cash
outflows of the supplying division plus
the contribution to the supplying
division from an outside sale.
C. The price usually set by an
absorption-costing calculation.
D. The price set by charging for
variable costs plus a lump sum or an
additional markup, but less than full
markup.

[104] Source: CIA 0595 III-96


Which of the following techniques would be
best for evaluating the management
performance of a department that is operated
as a cost center?
A. Return on assets ratio.
B. Return on investment ratio.
C. Payback method.
D. Variance analysis.

[101] Source: Publisher


Variable-cost-plus price is
A. The price on the open market.
B. The price representing the cash
outflows of the supplying division plus
the contribution to the supplying
division from an outside sale.
C. The price usually set by an
absorption-costing calculation.

[105] Source: CMA 0697 3-29


Listed below is selected financial
information for the Western Division of the
Hinzel Company for last year.
Amount
Account
(thousands)
??????????????????????????????????? ??????????
?
Average working capital
$ 625
General and administrative expenses
75
Net sales
4,000
Average plant and equipment
1,775

Cost of goods sold


3,525
If Hinzel treats the Western Division as an
investment center for performance
measurement purposes, what is the
before-tax return on investment (ROI) for
last year?

assembly line?
A. Supervisory salaries.
B. Materials.

A. 34.78%

C. Repairs and maintenance.

B. 22.54%

D. Depreciation on equipment.

C. 19.79%
D. 16.67%
[106] Source: CMA 0697 3-24
Residual income is a better measure for
performance evaluation of an investment
center manager than return on investment
because
A. The problems associated with
measuring the asset base are eliminated.

[109] Source: Publisher


Charlie's Service Co. is a service center.
For the month of June, Charlie's had the
following operating statistics:
Sales
$750,000
Operating income
25,000
Net profit after taxes
8,000
Total assets available
500,000
Shareholders' equity
200,000
Cost of capital
6%
Charlie's has a

B. Desirable investment decisions will


not be neglected by high-return
divisions.

A. Return on investment of 3.33%.

C. Only the gross book value of assets


needs to be calculated.

C. Return on investment of 6%.

B. Residual income of $(5,000).

D. Residual income of $(20,000).


D. The arguments about the implicit
cost of interest are eliminated.
[107] Source: Publisher
The following forecasted information is
available for a manufacturing division for
next year:
Amount
Category
(thousands)
???????????????????
???????????
Working capital
$ 1,800
Revenue
30,000
Plant and equipment
17,200
To establish a standard of performance for
the division's manager using the residual
income approach, four scenarios are being
considered. Scenario 1 assumes an imputed
interest charge of 15% and a target residual
income of $2,000,000. Scenario 2 assumes
an imputed interest charge of 12% and a
target residual income of $1,500,000.
Scenario 3 assumes an imputed interest
charge of 18% and a target residual income
of $1,250,000. Scenario 4 assumes an
imputed interest charge of 10% and a target
residual income of $2,500,000. Which of the
scenarios assumes the lowest maximum
cost?

[110] Source: CMA Samp Q3-2


Consider the following information for
Richardson Company for the prior year.
- The company produced 1,000 units and sold 900 units,
both as
budgeted.
- There were no beginning or ending work-in-process
inventories and no
beginning finished goods inventory.
- Budgeted and actual fixed costs were equal, all variable
manufacturing costs were affected by production volume
only, and all
variable selling costs were affected by sales volume only.
- Budgeted per unit revenues and costs were as follows:
Per unit
????????
Sales price
$100
Direct materials
30
Direct labor
20
Other variable manufacturing costs
10
Fixed selling costs
5
Variable selling costs
12
Fixed selling costs ($33,600 total)
4
Fixed administrative costs ($1,800 total) 2
The contribution margin earned by
Richardson for the prior year was
A. $25,200

A. Scenario 1.
B. $28,000
B. Scenario 2.
C. $31,500
C. Scenario 3.
D. $35,000
D. Scenario 4.
[108] Source: CMA 0691 3-26
If a manufacturing company uses
responsibility accounting, which one of the
following items is least likely to appear in a
performance report for a manager of an

[111] Source: Publisher


Assume Avionics Industrials reported at
year-end that operating income before taxes
for the year equaled $2,400,000. Long-term
debt issued by Avionics has a coupon rate
equal to 6%, and its cost of equity is 8%.

The book value of the debt currently equals


its fair value, and the book value of the
equity capital for Avionics is $900,000 less
than its fair value. Current assets are listed
at $2,000,000 and long-term assets equal
$9,600,000. The claims against those assets
are in the form of $1,500,000 in current
liabilities and $2,200,000 in long-term
liabilities. The income tax rate for Avionics
is 30 percent. What is the economic value
added (EVA)?

Revenue
30,000
Plant and equipment
17,200
If the imputed interest charge is 15% and
Webb wants to achieve a residual income
target of $2,000,000, what will costs have
to be in order to achieve the target?
A. $9,000,000
B. $10,800,000
C. $25,150,000

A. $731,240
D. $25,690,000
B. $948,760
C. $1,668,760
D. $1,680,000
[112] Source: Publisher
What is the weighted-average cost of capital
(WACC) to be used in the economic value
added (EVA) calculation?

[115] Source: Publisher


An effective managerial control system has
favorable motivational effects. Which of the
following are aspects of motivation?
Managerial
Goal
Effort
Congruence
?????????? ??????????
A.

A. 8%

No

No

No

Yes

Yes

Yes

Yes

No

B.
B. 8.89%
C. 9%

C.

D. 10%
D.
[Fact Pattern #5]
Dzyubenko Co. reported these data at
year-end:
Pre-tax operating income $ 4,000,000
Current assets
4,000,000
Long-term assets
16,000,000
Current liabilities
2,000,000
Long-term liabilities
5,000,000
The long-term debt has an interest rate of
8%, and its fair value equaled its book
value at year-end. The fair value of the
equity capital is $2 million greater than its
book value. Dzyubenko's income tax rate is
25%, and its cost of equity capital is 10%.
[113] Source: Publisher
(Refers to Fact Pattern #5)
The EVA is
A. $1,380,000

[116] Source: Publisher


Goal congruence is most likely to be
promoted when
A. A company uses a cost-based
transfer price.
B. A manager of a retail store is
charged imputed interest on inventory.
C. Percentage return on investment
rather than residual income is used to
measure managerial performance.
D. An annual accrual accounting
measure is used to evaluate a manager
who makes capital investment
decisions.

B. $1,620,000
C. $1,830,000

[117] Source: CIA 1195 II-36


To make goal setting effective and
worthwhile, the goals should be

D. $3,000,000
A. Just beyond what subordinates are
likely to reach.
[114] Source: CMA 0697 3-30
(Refers to Fact Pattern #5)
James Webb is the general manager of the
Industrial Product Division, and his
performance is measured using the residual
income method. Webb is reviewing the
following forecasted information for his
division for next year:
Amount
Category
(thousands)
???????????????????
???????????
Working capital
$ 1,800

B. Qualitative and approximate.


C. Based on superior performers'
output.
D. Specific, objective, and verifiable.
[118] Source: Publisher
The extent to which a manager can influence
organizational activities is

A. Authority.
B. Responsibility.

[122] Source: Publisher


MBO managers are most likely to believe
that employees

C. Accountability.

A. Dislike their work.

D. Controllability.

B. Avoid responsibility whenever


possible.

[119] Source: CMA 0693 3-21


Which one of the following organizational
policies is most likely to result in
undesirable managerial behavior?
A. Patel Chemicals sponsors television
coverage of cricket matches between
national teams representing India and
Pakistan. The expenses of such media
sponsorship are not allocated to its
various divisions.
B. Joe Walk, the chief executive officer
of Eagle Rock Brewery, wrote a
memorandum to his executives stating,
"Operating plans are contracts, and they
should be met without fail."
C. The budgeting process at Madsen
Manufacturing starts with operating
managers providing goals for their
respective departments.
D. Fullbright Lighting holds quarterly
meetings of departmental managers to
consider possible changes in the
budgeted targets due to changing
conditions.
[120] Source: Publisher
An advantage of participatory budgeting is
that it

C. Work best when threatened with


punishment.
D. Are self-motivated.
[123] Source: Publisher
A budget is often the result of a
management-by-objectives (MBO) program.
A characteristic of MBO is
A. Development of a single measure of
employee performance.
B. Statement of objectives in general
terms.
C. Establishment of objectives through
both top-down and bottom-up
processes.
D. A flexible time frame for
achievement of objectives.
[124] Source: Publisher
A company has a compensation system for
its managers based on an MBO approach.
The essential premise of MBO is that
A. Compensation should be based on
qualitative factors.

A. Minimizes the cost of developing


budgets.

B. Employees should be concerned with


routine matters, and managers should
attend to exceptions.

B. Reduces the effect on the budgetary


process of employee biases.

C. Employees should participate in


setting goals.

C. Yields information known to


management but not to employees.

D. Managers should establish goals for


their employees.

D. Encourages acceptance of the budget


by employees.
[121] Source: CMA 0693 3-26
Which one of the following will not occur in
an organization that gives managers
throughout the organization maximum
freedom to make decisions?
A. More effective solutions to
operational problems.
B. Individual managers regarding the
managers of other segments as they do
external parties.
C. Two divisions of the organization
having competing models that aim for
the same market segments.
D. Delays in securing approval for the
introduction of new products.

[125] Source: Publisher


Which of the following is a weakness of
using MBO for motivating employees?
A. Employees may believe that they
control situations that arise in the
company.
B. Employees may be forced to place
too much emphasis on quantitative
factors.
C. Employees may participate too much
in the goal-setting process.
D. Employees may become too trusting
and too dependent on the "team" to get
things done.
[126] Source: Publisher
Managerial performance may be measured
in many ways. For example, an internal
nonfinancial measure is

A. Market share.

D. Improvement in managerial morale


and the potential for behavioral change.

B. Delivery performance.
C. Customer satisfaction.

[130] Source: Publisher


Common costs are

D. Manufacturing lead time.


A. Direct costs.
[127] Source: Publisher
Many forms of performance feedback are
based on accounting information. For
example, a divisional manager may be
evaluated based on return on investment
(income investment). One step in the
process of developing a performance
measure based on accounting information is
to determine the basis for stating the
measure in terms of dollars. Thus, if ROI is
the chosen measure, and investment is
defined as total assets, comparability issues
are most likely to arise when the attribute
used to calculate total assets is

B. Current costs.
C. Controllable costs.
D. Indirect costs.
[131] Source: Publisher
Managers are most likely to accept
allocations of common costs based on
A. Cause and effect.
B. Ability to bear.

A. Current cost.

C. Fairness.

B. Current disposal price.

D. Benefits received.

C. Historical cost.
D. Present value.
[128] Source: Publisher
An organization's managerial
decision-making model for capital budgeting
is based on the net present value of
discounted cash flows. The same
organization's managerial performance
evaluation model is based on annual
divisional return on investment. Which of
the following is true?
A. Divisional managers are likely to
maximize the measures in the
decision-making model.
B. Divisional managers are likely to
maximize the measures in the
performance evaluation model.
C. The manager has an incentive to
accept a project with a positive net
present value that initially has a
negative effect on net income.
D. The use of models with different
criteria promotes goal congruence.

[132] Source: Publisher


A large corporation allocates the costs of its
headquarters staff to its decentralized
divisions. The best reason for this allocation
is to
A. More accurately measure divisional
operating results.
B. Improve divisional managements'
morale.
C. Remind divisional managers that
common costs exist.
D. Discourage any use of central
support services.
[133] Source: Publisher
Which budget is most likely to facilitate
variance analysis?
A. Fixed budget.
B. Static budget.
C. Continuous budget.
D. Flexible budget.

[129] Source: Publisher


In a well-run organization, a manager may
have responsibility for activities that (s)he
cannot significantly influence. This
arrangement may be justified because of the

[134] Source: Publisher


Using the balanced scorecard approach, an
organization evaluates managerial
performance based on

A. Manager's knowledge about the


activities and the potential for
behavioral change.

A. A single ultimate measure of


operating results, such as residual
income.

B. Manager's knowledge about the


activities but not the potential for
behavioral change.

B. Multiple financial and nonfinancial


measures.
C. Multiple nonfinancial measures only.

C. Improvement in managerial morale,


effort, and performance.

D. Multiple financial measures only.

their culpability.
[135] Source: CIA 0594 III-50
Which of the following is not an advantage
of teamwork compared with work
performed by individuals?
A. Teams provide support to the team
members.
B. Teams make decisions that are more
easily accepted.
C. Teams provide a clear link between
effort and outcome.

D. Focusing the discussion on the


offending behavior instead of the
offender.
[140] Source: CIA 0594 III-83
An organization's management perceives the
need to change fundamentally. Which of the
following factors is management least likely
to change?
A. The organization's members.
B. The organization's structure.

D. Teams control and discipline


members.

C. The organization's environment.


D. The organization's technology.

[136] Source: CIA 1193 III-2


Which of the following is most likely to
provide for the continual development of
managerial skills?
A. Organizational orientation sessions.
B. Job rotation.
C. Role playing.
D. Liaison committees.

[141] Source: CIA 1194 II-26


While conducting fieldwork, a strong
conflict arises between two of your
subordinates regarding possible scope
expansion. You draw their attention to their
shared views, downplaying the issues of
contention. This technique for resolving
conflict is called
A. Superordinate goals.
B. Smoothing.

[137] Source: CIA 0594 III-81


Which of the following would not be
considered a conflict trigger?

C. Problem solving.
D. Compromise.

A. Ambiguous jurisdictions.
B. Competition for scarce resources.
C. Status differential.
D. Superordinate goals.
[138] Source: CIA 0594 II-9
Which of the following is not an example of
positive reinforcement of behavior?
A. Paying a bonus to employees who
had no absences for any four-week
period.
B. Giving written warnings after only
every other absence.
C. Assigning a mentor to each
employee.
D. Having a lottery every month where
10% of the employees with no absences
receive a $200 bonus.

[142] Source: CIA 0595 II-32


Upon completing an audit of a major
division of an international chemical
manufacturer, the auditor noted that the
division seemed to focus almost solely on
the attainment of profits -- with little
attention paid to other issues. The auditor
had noted that the company's mission
statement emphasized generating returns to
the stockholders. The profit emphasis was
consistent with the mission statement. This
is an example of
I. A goal attainment approach in which the ends are the
focus
of attention and the means are evaluated only for their
promotion of the ends
II. An approach that is best suited for a company or division
at
the formalization or initialization phase of its business
III. An effective implementation of a strategic constituencies
approach because the organization is clearly focused on
developing returns for its stockholders
A. I only.
B. I and II only.

[139] Source: CIA 0594 III-89


The process of disciplining employees is
made less effective by:

C. III only.
D. I, II, and III.

A. Stating the offending behavior


specifically.
B. Postponing the start of disciplinary
procedures.
C. Permitting employees to challenge

[143] Source: CIA 0595 II-35


The manager faces two potentially
conflicting trends when formulating the
presentation of audit reports: (A) the
information age and the increase in

information available for decision making;


and (B) the concept of "bounded
rationality," which pertains to the ability of
the individuals to process data and make
decisions. Which of the following
approaches to developing a report would be
consistent with the notion of bounded
rationality?
A. Identify the information most
available to encourage the user to make
an optimal decision.
B. Identify the problem, but let the
decision maker independently search
the databases for the problem solution.
C. Present data in graphic form to focus
on the major, more simple aspects of
complex problems.
D. Suggest that a short time limit be
placed on the time to make a decision
so that the decision maker will
concentrate on taking effective action in
a timely fashion.
[144] Source: CIA 0595 II-34
There has been an increased emphasis on
group decision making in organizations.
Which of the following statements has been
found to hold true in studies of individual
decision making as compared to group
decision making? Individual decision
making tends to

based on this observation?


A. This supervisor is too lenient in
performance ratings.
B. The performance appraisal forms
probably have too many leading
questions.
C. There may be a systematic bias in the
way individuals are being rated.
D. This supervisor is trying too hard to
treat everyone the same.
[147] Source: CIA 1195 II-16
Which one of the following techniques is not
generally recognized as an effective conflict
resolution technique for management to use
in a dispute between employees?
A. Command - management resolves the
conflict by using its formal authority.
B. Competition - management
encourages the parties to seek their own
interests, regardless of the effect on
each other.
C. Reorganization - management
transfers one of the disputants to another
department.
D. Compromise - management
persuades each party to make
concessions.

A. Be more conservative.
B. Evaluate more complete information.
C. Generate more alternatives.
D. Be more time consuming.
[145] Source: CIA 1195 II-1
Which of the following statements about
group decision making is generally
considered false?
A. There is a lack of responsibility for
group decisions.

[148] Source: CIA 1195 II-18


A manager must design a program to help
subordinates learn about a new procedure
that an organization has implemented and is
considering both classroom lecture and
comprehensive on-the-job (experiential)
training approaches. Which one of the
following is a disadvantage of the
traditional classroom lecture format as a
training method when compared to learning
by experience?
A. It is more expensive.
B. It takes longer.

B. Group decision making is almost


always less efficient than individual
decision making.
C. Although group members frequently
have diverse views about a decision,
each member's desire to be accepted by
the group often restrains open
disagreement.
D. Group decision making tends to be
less creative than individual decision
making.
[146] Source: CIA 1195 II-15
A manager notices that a particular
supervisor's performance appraisal ratings
for any given subordinate tend to be
consistently high, consistently low, or
consistently in the middle across all
performance dimensions. Which one of the
following is the best conclusion to draw

C. It is difficult to go in-depth on a
given topic.
D. Trainees do not easily retain what
they learn in the classroom.
[149] Source: CIA 1195 II-33
Which one of the following statements about
quality circles is false?
A. A quality circle is typically
comprised of a group of 8 to 10
subordinates and supervisors.
B. Part of the quality circle concept
includes teaching participants
communication skills, quality strategies,
and problem analysis techniques.
C. Quality circles meet on the company

premises and on company time.


D. The quality circle has the final
control over implementation of
recommended solutions.
[150] Source: CIA 1195 II-36
To make goal-setting effective and
worthwhile, the goals should be
A. Just beyond what subordinates are
likely to reach.

of group decision making as compared to


individual decision making?
A. Groups obtain an increased degree
of acceptance of a solution so that it
may be more easily implemented.
B. Group decision making is consistent
with democratic methods.
C. Group members bring more complete
information and knowledge into the
decision process.

B. Qualitative and approximate.


C. Based on superior performers'
output.

D. Group members avoid expressing


opinions that deviate from what appears
to be the group consensus.

D. Specific, objective, and verifiable.


[151] Source: CIA 1194 II-28
The least effective method for an employee
to learn computer skills is
A. Classroom training using equipment.
B. Video training.
C. Apprenticeships.
D. Case analysis.
[152] Source: CIA 1196 II-38
Which of the following statements is correct
with respect to a change in values?
A. Values are neither stable nor
enduring.
B. The process of questioning values
will result in a change.

[155] Source: CIA 0596 II-20


The best way a new employee can learn
about the company's corporate culture
would be to
A. Seek the advice of a more
experienced person who was hired at
the same time to learn about how the
company works.
B. Read professional literature and
journals to ascertain how experts view
the company.
C. Watch the behavior of others to
determine what works and what does
not.
D. Ask several managers to explain
how their behavior is consistent with
the organizational culture.

C. Values are not fixed, and when they


change, they change quickly.

[156] Source: CIA 0596 II-32


The most effective way for a manager to
delegate a task to a staff member would be
to

D. Values are established in early years


and are unlikely to change.

A. Define the desired outcome and the


approach precisely and in writing.

[153] Source: CIA 0596 II-33


A manager discovers by chance that a newly
hired auditor has strong beliefs that are very
different from the manager's and from those
of most of the audit staff. The manager's best
course of action would be to
A. Facilitate the reassignment of the
new hire as quickly as possible before
this situation becomes disruptive.
B. Ask the rest of the team for their
reaction and act according to the group
consensus.
C. Take no action unless the new hire's
behavior is likely to cause harm to the
organization.
D. Try to counsel the new hire into
more reasonable beliefs.
[154] Source: CIA 1196 II-37
Which of the following is not an advantage

B. Define the desired outcome


precisely, discuss possible approaches
with the staff auditor, and reach
agreement on the approach to be taken.
C. Let the staff member try to perform
the task for a defined period of time and
then meet to critique the approach,
clarifying the assignment as needed.
D. Give the assignment in very general
terms, have the staff member develop
the desired outcome and approach, and
then review and critique the staff
member's decisions.
[157] Source: CIA 1196 II-26
If a supervisor uses a supportive
management approach, evidenced by
positive feelings and concern for
subordinates, a problem might result
because
A. An approach based on pure power
makes it difficult to motivate staff.

B. This approach depends on material


rewards for the worker.

As a conflict resolution strategy, optimizing


(or a win-win strategy) is most appropriate
when

C. This approach depends on people


who want to work, grow, and achieve.

A. The benefits being contested cannot


be changed.

D. The manager must believe in the


teamwork approach.

B. The relationship between the parties


is likely to continue.

[158] Source: CIA 1196 II-27


Some behavioral models stress employee
participation as a key to motivation. A
limitation of the participative approach is
A. Workers are intrinsically lazy and
must be driven.
B. A number of dissatisfiers must be
present in order for the approach to
work.
C. It is difficult to elicit the
participation of all employees.
D. Unresolvable conflicts arise when a
mature, capable, creative person joins a
structured, demanding, and limiting
organization.

C. People are deeply committed to


established habits and patterns.
D. Time is scarce and the manager's
patience is wearing thin.
[162] Source: CIA 1196 II-19
The behavioral science literature identifies
diffusion as an effective approach to
resolving conflict. A manager effectively
using diffusion in working with a
confrontational employee would
A. Set aside critical issues temporarily
and try to reach agreement on less
controversial issues first.
B. Emphasize differences between the
parties.
C. Avoid the conflict situation.

[159] Source: CIA 1196 II-32


A production worker in a plant often speaks
for the entire workforce when problems
arise between labor and management.
Although this individual has the same level
of authority and expertise as his co-workers,
he seems to possess a degree of power that
others do not have. What type of power does
this individual apparently have?
A. Coercive.
B. Referent.
C. Expert.
D. Legitimate.

D. Identify the sources of conflict and


address them directly.
[163] Source: CIA 1196 II-33
During a performance appraisal, the
manager experienced difficulty obtaining
required information from a specific
employee. The manager requested a private
meeting with the employee for the purpose
of identifying the problem and resolving the
difficulty through open discussion. Which
conflict management technique was the
auditor applying?
A. Problem solving.
B. Expansion of resources.

[160] Source: CIA 1196 II-34


A manager believes that positive
reinforcement is the most appropriate way
to deal with professional staff of the type
working in his/her department. Which of the
following actions demonstrates the principle
of positive reinforcement?
A. An employee is given a 2-day
suspension without pay if errors exceed
a predefined level.
B. Employees are praised when the
detected error rate in their work stays
below a pre-defined level.

C. Authoritative command.
D. Altering the human variable.
[164] Source: CIA 0596 II-27
A worker receives outstanding performance
evaluations from his first three supervisors.
He tries hard to please his fourth supervisor,
but the more he accomplishes, the more
critical his new supervisor becomes. The
new supervisor is generally considered a
marginal performer. Which of the following
is the most likely source of the problem?

C. Time budgets, which have forced


employees to rush and consequently
make errors, are eliminated.

A. The supervisor's insecurity.

D. Employees are not required to work


overtime if errors detected in their
work stay below a predefined level.

C. The work environment.

[161] Source: CIA 0596 II-22

B. The worker's performance.

D. The job standards.


[165] Source: CIA 1196 II-28

Which of the following criteria for


measuring the quality of employee
performance would be appropriate for use
with a group of professionals, such as a
college faculty?
I. Quantity of output
II. Quality of output
III. Cost
IV. Timeliness of output
V. Capital requirements
VI. Revenue produced
A. I, II, and III only.
B. I, II, and IV only.
C. III, V, and VI only.

C. Inventory increases or decreases.


D. Changes in fixed manufacturing
overhead.
[Fact Pattern #6]
GWW Company incurred the following
manufacturing costs in its first month of
operations.
Raw materials
$10,000
Direct labor
20,000
Variable factory overhead
20,000
Fixed factory overhead
30,000
Production totaled 2,000 units. Sales amounted to 1,900
units at a
price of $100 each.

D. I, II, III, IV, V, and VI.


[166] Source: CIA 1196 II-30
Which of the following is not a good
example of a way to evaluate the quality of
an individual's work?
A. Evaluate the task outcome to
determine if the job gets done.

[170] Source: Publisher


(Refers to Fact Pattern #6)
The ending inventory under the variable
costing method is
A. $1,500.
B. $2,500.
C. $4,000.

B. Use multiple evaluators to increase


reliability.
D. $10,000.
C. Use multiple criteria because every
employee is expected to do a number of
things well.
D. Evaluate the worker's traits, such as
attitude, intelligence, or reliability.

[171] Source: Publisher


(Refers to Fact Pattern #6)
The ending inventory under the absorption
costing method is
A. $1,500.

[167] Source: Publisher


Net income is lower under variable costing
than under absorption costing when

B. $2,500.
C. $4,000.

A. Production increases from the


previous period.

D. $10,000.

B. Production exceeds sales.


C. Production equals sales.
D. Production is less than sales.

[172] Source: Publisher


(Refers to Fact Pattern #6)
Absorption costing profits will exceed
variable costing profits by
A. $1,500.

[168] Source: Publisher


If a company uses variable costing instead
of absorption costing, it will always report
a
A. Higher income.

B. $4,000.
C. $30,000.
D. $0.

B. Lower income.
C. Higher finished goods inventory.

[173] Source: Publisher


If inventory quantities increase during a
period,

D. Lower finished goods inventory.


A. Variable costing profits will exceed
absorption costing profits.
[169] Source: Publisher
Under variable costing, profit fluctuates in
relation to
A. Sales.
B. Production.

B. Absorption costing profits will


exceed variable costing profits.
C. Variable costing profits will equal
absorption costing profits.
D. Variable costing will show a higher
inventory value than absorption costing.

A. Variable manufacturing overhead


manufacturing overhead costs.
[174] Source: Publisher
Advocates of the variable costing method
believe that fixed overhead costs should be
expense as incurred because
A. Product costs will occur regardless

B. Fixed manufacturing overhead costs.


C. All overhead costs.
D. All manufacturing costs.

of production levels.
B. Allocation of fixed production costs
is arbitrary.
C. Production costs are uncontrollable
and should not be charged to
production.
D. Period costs are not necessary to
produce a product.

[179] Source: Publisher


If unit costs remain unchanged and sales
volume and sales price per unit both
increase from the preceding period when
operating profits were earned, operating
profits must
A. Increase under the absorption costing
method.
B. Increase under the variable costing
method.

[175] Source: Publisher


Although the absorption costing method is
ordinarily required by GAAP, a company
may also want to prepare statements under
the variable costing method to
A. Report a greater income.
B. Better evaluate the activities of
production managers.
C. Better evaluate sales personnel.

C. Decrease under the absorption


costing method.
D. Decrease under the variable costing
method.
[180] Source: Publisher
Which of the following is NOT an
advantage of using variable costing for
internal reporting purposes?

D. Reduce income taxes.


[176] Source: Publisher
Under the variable costing method of cost
allocation,
A. Variable manufacturing overhead is
treated as a period cost.
B. Variable manufacturing overhead is
expensed as incurred.

A. Fixed costs are reported at incurred


values, not absorbed values, thus
improving control over those costs.
B. Profits are directly influenced by
changes in sales volume.
C. The impact of fixed costs on profits
is emphasized.
D. Total costs may be overlooked when
evaluating profits.

C. Variable direct labor costs are


expensed as period costs.
D. Variable manufacturing overhead is
inventoried as a production cost.

[181] Source: Publisher


Which of the following is an advantage of
the variable costing method?
A. Marginal contribution is emphasized.

[177] Source: Publisher


As compared with total absorption costing
profit over the entire life of a company, total
variable costing profit will
A. Be less.

B. The distinction between fixed and


variable costs is often too difficult to
determine in practice.
C. Financial statements might confuse
users who are accustomed to absorption
costing statements.

B. Be greater.
C. Be equal.
D. Be substantially greater or less
depending upon external factors.
[178] Source: Publisher
Absorption costing inventories all
manufacturing overhead costs. Variable
costing expenses which of the following as
period costs?

D. Emphasis on variable costs may


cause managers to ignore fixed costs.
[182] Source: Publisher
Contribution margin is defined as the
difference between
A. Variable costing and absorption
costing net income.
B. Sales and total variable costs.

C. Fixed and variable manufacturing


costs.
D. Sales and fixed manufacturing costs.
[183] Source: Publisher
A company's flexible budget shows an
expected variable delivery expense of
$160,000 when sales are 50,000 units. If
sales total 52,000 units, and the actual
delivery expense is $163,000, what will be
the given sales variance?
A. $3,000 unfavorable.

[187] Source: Publisher


Sheila is a financial manager who has
discovered that her company is violating
environmental regulations. If her immediate
superior is involved, her appropriate action
is to
A. Do nothing since she has a duty of
loyalty to the organization.
B. Consult the audit committee.
C. Present the matter to the next higher
managerial level.
D. Confront her immediate superior.

B. $3,400 unfavorable.
C. $3,000 favorable.
D. $3,400 favorable.
[184] Source: Publisher
A company's flexible budget shows an
expected fixed cost of $100,000 for
straight-line depreciation when sales total
50,000 units. If sales total 52,000 units, and
the actual cost of depreciation is $103,000,
what will be the budget variance?
A. $1,000 favorable.

[188] Source: Publisher


Corporate social responsibility is
A. Effectively enforced through the
controls envisioned by classical
economics.
B. Defined as the obligation to
shareholders to earn a profit.
C. More than the obligation to
shareholders to earn a profit.
D. Defined as the obligation to serve
long-term, organizational interests.

B. $3,000 favorable.
[189] Source: Publisher
C. $1,000 unfavorable.
D. $3,000 unfavorable.
[185] Source: CMA Samp Q3-12
To meet Zylon Corporation's overall
objectives, the Frame Division has just
initiated a program to increase sales by
improving the manufacturing quality of its
products. The most appropriate management
level to be responsible for this program is
the
A. Sales manager.

A common argument against corporate


involvement in socially responsible
behavior is that
A. It encourages government intrusion
in decision making.
B. As a legal person, a corporation is
accountable for its conduct.
C. It creates goodwill.
D. In a competitive market, such
behavior incurs costs that place the
company at a disadvantage.

B. Production manager.
C. Division president.
D. Receiving and inspection manager.
[186] Source: Publisher
If a financial manager/management
accountant has a problem in identifying
unethical behavior or resolving an ethical
conflict, the first action (s)he should
normally take is to
A. Consult the board of directors.
B. Discuss the problem with his/her
immediate superior.
C. Notify the appropriate law
enforcement agency.
D. Resign from the company.

[190] Source: Publisher


The IMA Code of Ethics requires a financial
manager/management accountant to follow
the established policies of the organization
when faced with an ethical conflict. If these
policies do not resolve the conflict, the
financial manager/management accountant
should
A. Consult the board of directors
immediately.
B. Discuss the problem with the
immediate superior if (s)he is involved
in the conflict.
C. Communicate the problem to
authorities outside the organization.
D. Contact the next higher managerial
level if initial presentation to the
immediate superior does not resolve the
conflict.

[191] Source: Publisher


Financial managers/management accountants
are obligated to maintain the highest
standards of ethical conduct. Accordingly,
the IMA Code of Ethics explicitly requires
that they
A. Obtain sufficient competent evidence
when expressing an opinion.
B. Not condone violations by others.
C. Comply with generally accepted
auditing standards.
D. Adhere to generally accepted
accounting principles.
[192] Source: Publisher
Integrity is an ethical requirement for all
financial managers/management accountants.
One aspect of integrity requires
A. Performance of professional duties
in accordance with applicable laws.
B. Avoidance of conflict of interest.
C. Refraining from improper use of
inside information.
D. Maintenance of an appropriate level
of professional competence.
[193] Source: Publisher
A financial manager/management accountant
discovers a problem that could mislead
users of the firm's financial data and has
informed his/her immediate superior. (S)he
should report the circumstances to the audit
committee and/or the board of directors only
if
A. The immediate superior, who reports
to the chief executive officer, knows
about the situation but refuses to correct
it.
B. The immediate superior assures the
financial manager/management
accountant that the problem will be
resolved.
C. The immediate superior reports the
situation to his/her superior.
D. The immediate superior, the firm's
chief executive officer, knows about the
situation but refuses to correct it.
[194] Source: Publisher
In which situation is a financial
manager/management accountant permitted
to communicate confidential information to
individuals or authorities outside the firm?
A. There is an ethical conflict and the
board has refused to take action.
B. Such communication is legally
prescribed.

C. The financial manager/management


accountant knowingly communicates the
information indirectly through a
subordinate.
D. An officer at the financial
manager/management accountant's bank
has requested information on a
transaction that could influence the
firm's stock price.

CMA PART 3A
BEHAVIOURAL ISSUES
ANSWERS
[1] Source: Publisher
Answer (A) is incorrect because a
top-down budget is generated by top
management and distributed to
(imposed on) lower-level managers.
Answer (B) is incorrect because a
bottom-up budget is generated by
lower-level management and
aggregated as it moves through the chain
of command.
Answer (C) is correct. The management
by objectives (MBO) approach is a
procedure in which a subordinate and a
supervisor agree on goals and the
methods of achieving them and develop
a plan in accordance with that
agreement. The subordinate is then
evaluated with reference to the plan at
the end of the plan period.
Answer (D) is incorrect because the
management by exception approach
uses measurable standards and
deviations therefrom to determine when
management action is needed.
[2] Source: CMA 1292 3-30
Answer (A) is incorrect because
reciprocal cost allocation does nothing
to enhance goal congruence.

[3] Source: CMA 0691 3-24


Answer (A) is correct. The ROI
calculation divides return (income) by
invested capital. Invested capital may
be defined in various ways, for
example, as total assets available, total
assets employed (assets that are idle,
such as vacant land, are excluded),
working capital plus other assets
(current liabilities are deducted from
total assets to exclude the assets
provided by short-term creditors), and
stockholders' equity (a portion of
long-term liabilities must be allocated
to determine the manager's resource
base). Total assets available is the
measure of invested capital that
assumes the manager will use all assets
without regard to financing.
Answer (B) is incorrect because use of
total assets employed assumes that
higher-level management has not
required the manager to assume
responsibility for idle assets.
Answer (C) is incorrect because
deducting current liabilities effectively
excludes assets financed by short-term
credit.
Answer (D) is incorrect because
shareholders' equity is determined by
deducting total liabilities from total
assets. Thus, this measure of the
investment base also considers the issue
of financing.
[4] Source: CMA 0691 3-29

Answer (B) is incorrect because


reciprocal cost allocation, zero-base
budgeting, and standard costing are not
designed to enhance goal congruence.
Answer (C) is incorrect because
imposed budgeting and activity-based
costing are not designed to enhance goal
congruence.
Answer (D) is correct. Transfer prices
based on cost promote goal congruence
by ensuring that purchases are made at
the lowest cost for the entity as a whole.
Management-by-objective (MBO)
performance evaluation can also be a
goal congruence tool. MBO is a
behavioral, communications-oriented,
responsibility approach to employee
self direction. MBO is based on the
philosophy that employees want to
work hard if they only know what is
expected, that employees like to know
what their job actually is, and that
employees are capable of self direction
and self motivation. The key is to
coordinate managers' goals with the
overall goals of the organization.
Participation in budgeting by those
affected likewise encourages goal
congruence because those who take part
in the budget process are likely to
support the outcome.

Answer (A) is incorrect because total


assets employed does not reflect
financing decisions.
Answer (B) is incorrect because
deducting current liabilities from
current assets to arrive at working
capital reflects the control that the
manager of the responsibility center
ordinarily has over short-term credit
transactions.
Answer (C) is incorrect because total
assets available does not reflect
financing decisions.
Answer (D) is correct. Stockholders'
equity equals total assets minus total
liabilities. The latter include short-term
liabilities incurred at operating levels
of the organization and long-term
liabilities resulting from financing
decisions made by top management.
Accordingly, the investment base used
to measure the performance of a
manager may reflect the incurrence of
liabilities over which (s)he had no
control. A second problem is that the
allocation of long-term liabilities
among divisions or segments may be
somewhat arbitrary.

[5] Source: CMA 1291 3-9


Answer (A) is incorrect because an
investment center has authority not only
over costs and revenues, but also
capital invested.
Answer (B) is correct. An investment
center is responsible for revenues,
expenses, and invested capital. Return
on investment is usually the key
performance measure of an investment
center.
Answer (C) is incorrect because an
investment center also has authority
over revenues and invested capital.
Answer (D) is incorrect because a
service center supports other
organizational units.
[6] Source: CMA 1292 3-24
Answer (A) is incorrect because an
investment base is need to calculate
residual income.
Answer (B) is correct. The residual
income method calculates the excess of
the return on an investment over a
targeted amount equal to an imputed
interest charge on invested capital. The
rate used is usually the weighted
average cost of capital. Residual
income may be preferable to ROI
because an enterprise will benefit from
expansion as long as residual income is
earned. Using a ROI, expansion might
be rejected if it lowered ROI even
though residual income would increase.
Thus, the residual income method
promotes the congruence of a manager's
goal with those of the enterprise.
Actions that tend to benefit the company
will also tend to improve the measure
of the manager's performance.
Answer (C) is incorrect because ROI is
more widely used than residual income.
Answer (D) is incorrect because both
measures consider the same items.

performance report showing actual


costs incurred against budgeted costs
permits evaluation of a manager and the
area for which (s)he is responsible.
Answer (D) is incorrect because
activity-based costing is a means of
allocating overhead.
[8] Source: CMA 0694 3-28
Answer (A) is correct. A well-designed
responsibility accounting system
establishes responsibility centers within
an organization. Managerial
performance should be evaluated only
on the basis of those factors
controllable by the manager. Managers
may control revenues, costs, and/or
investment activities. The responsibility
system should induce management
performance that adheres to overall
company objectives. Charging the costs
of a rush order to the sales manager
who authorized the job creates an
incentive for that individual to minimize
such costs.
Answer (B) is incorrect because
functional accounting accumulates costs
and assets for each service provided or
function performed, without necessarily
assigning responsibility for such costs.
Answer (C) is incorrect because a
transfer-pricing system charges one
segment of an organization for goods
and services that are provided by
another segment within the organization.
Answer (D) is incorrect because
contribution accounting emphasizes
variable costs and their relationship
with revenues, but disassociates fixed
costs from the departments responsible.
[9] Source: CMA 1294 3-22
Answer (A) is incorrect because the
manager of an assembly line is likely to
be responsible for the salaries of
supervisors, which is to some degree
controllable by the manager.

[7] Source: CMA 0694 3-27


Answer (A) is incorrect because
transfer prices are amounts charged by
one segment of an organization for
goods or services provided to another
segment.
Answer (B) is incorrect because
flexible budgeting is simply a series of
budgets for varying levels of activity.
Answer (C) is correct. A well-designed
responsibility accounting system
establishes responsibility centers within
an organization. Managerial
performance should be evaluated only
on the basis of those factors
controllable by the manager. Managers
may control revenues, costs, and/or
investment activities. A departmental

Answer (B) is incorrect because the


manager of an assembly line is likely to
be responsible for the materials, which
is to some degree controllable by the
manager.
Answer (C) is incorrect because the
manager of an assembly line is likely to
be responsible for the repairs and
maintenance, which is to some degree
controllable by the manager.
Answer (D) is correct. Responsibility
accounting holds managers responsible
only for factors under their control. The
depreciation of equipment will
probably not appear on the performance
report of an assembly-line manager
because the manager usually has no
control over the investment in the

equipment.
[10] Source: Publisher
Answer (A) is incorrect because dollar
sales do not give a measure of operating
performance based on resources
required.
Answer (B) is incorrect because net
income does not give a measure of
operating performance based on
resources required.
Answer (C) is incorrect because profit
percentages do not give a measure of
operating performance based on
resources required.
Answer (D) is correct. Each investment
center of a business should be evaluated
based upon return on investment to
judge operating performance. ROI is
comparable to calculations made both
within and without a particular
organization. Management may review
the investment opportunities available
within or without the firm. In essence,
net income is stated as a proportion of
investment capital (resources required).
[11] Source: Publisher
Answer (A) is incorrect because I NI
is the reciprocal of return on
investment.
Answer (B) is correct. The first term in
the formula for return on investment is
the number of times investment capital
has turned over through the sales
mechanism (S I). This amount is
multiplied by the net income expressed
as a percentage of sales (NI S) to give
the return on investment (NI I). The
basic formula for return on investment
is therefore
(S I) x (NI S) = NI I.
Answer (C) is incorrect because sales
must be divided by investment (S I) to
determine the number of times
investment capital has turned over,
which results in NI I as return on
investment.
Answer (D) is incorrect because net
income must be expressed as a
percentage of sales (NI S), which
results in NI I as return on investment.

Answer (C) is incorrect because


increasing sales volume while holding
other factors constant improves return
on investment.
Answer (D) is correct. ROI equals net
income divided by investment.
Increasing net income (e.g., by
decreasing expenses or by increasing
prices or sales volume) or decreasing
the investment base improves ROI.
Hence, any of the actions listed
increases the return on investment.
Management and the accounting
profession are very concerned with
classification of expenses and assets
and other decisions involving the
accounting for these items to achieve a
proper calculation of return on
investment.
[13] Source: CMA 0684 4-9
Answer (A) is incorrect because ROI
(NI I) will decrease if investment
(total assets) increases while NI
remains the same.
Answer (B) is correct. The basic
formula for return on investment is (S
I) x (NI S) = NI I. If NI increases
(because expenses have decreased) by
the same amount that I increases
(because total assets have increased),
ROI will increase. For example, if NI
equals 100 and I equals 1,000, the ROI
is 10%. An increase of 20 in each term
raises the ROI to 11.8%.
Answer (C) is incorrect because a
reduction in NI causes a decrease in
ROI if investment is constant.
Answer (D) is incorrect because ROI
(NI I) does not change if the company
originally had a net profit and the net
profit on sales (sales - expenses)
increases by the same percentage as the
total assets.
[14] Source: Publisher
Answer (A) is incorrect because equal
goals should not be set owing to
differences in competitive environment,
the strategic goals of the firm, and risk.
Answer (B) is incorrect because using
greater divisional ROIs in the less
profitable divisions to motivate those
divisions to achieve higher ROIs would
not necessarily improve divisional
performance.

[12] Source: Publisher


Answer (A) is incorrect because
decreasing expenses or assets while
holding other factors constant improves
return on investment.

Answer (C) is incorrect because lower


divisional ROIs in more profitable
divisions in which motivation is
unnecessary would likely suboptimize
divisional performance.

Answer (B) is incorrect because


increasing selling prices while holding
other factors constant improves return
on investment.

Answer (D) is correct. Each division


within a firm should have an ROI based
on the strategic goals of the firm
consistent with its competitive

environment.
[15] Source: Publisher
Answer (A) is incorrect because
motivation is the desire and the
commitment to achieve a specific goal.

some appropriate amount.


Answer (D) is incorrect because
different prices recorded by the buying
and selling divisions is characteristic of
a dual-pricing policy.
[18] Source: Publisher

Answer (B) is correct. Goal congruence


is agreement on the goals of the
organization and/or the segment by both
supervisors and subordinates.
Performance is assumed to be
optimized when there is an
understanding that personal and
segmental goals be consistent with
those of the organization.
Answer (C) is incorrect because
autonomy is the extent to which
individuals have the authority to make
decisions.
Answer (D) is incorrect because
managerial effort is the extent of the
attempt to accomplish a specific goal.
[16] Source: Publisher
Answer (A) is incorrect because the
retail price is the definition of the
market price, assuming an arm's-length
transaction.
Answer (B) is correct. At this price, the
supplying division is indifferent as to
whether it sells internally or externally.
Outlay cost plus opportunity cost
therefore represents a minimum
acceptable price for a seller. However,
no transfer price formula is appropriate
in all circumstances.
Answer (C) is incorrect because full
cost is the price usually set by an
absorption-costing calculation.
Answer (D) is incorrect because the
variable-cost-plus price is the price set
by charging for variable costs plus a
lump sum or an additional markup, but
less than full markup.
[17] Source: Publisher
Answer (A) is incorrect because the
price on the open market is the
definition of the market price.
Answer (B) is incorrect because outlay
cost plus opportunity cost is the price
representing the cash outflows of the
supplying division plus the contribution
to the supplying division from an
outside sale.
Answer (C) is correct. The
variable-cost-plus price is the price set
by charging for variable cost plus either
a lump sum or an additional markup but
less than the full markup price. This
permits top management to enter the
decision process and dictate that a
division transfer at variable cost plus

Answer (A) is incorrect because the


market price is the price on the open
market.
Answer (B) is incorrect because the
outlay cost plus opportunity cost is the
price representing the cash outflows of
the supplying division plus the
contribution to the supplying division
from an outside sale.
Answer (C) is correct. Full-cost price
is the price usually set by an
absorption-costing calculation and
includes materials, labor, and a full
allocation of manufacturing O/H. This
full-cost price may lead to
dysfunctional behavior by the supplying
and receiving divisions, e.g.,
purchasing from outside sources at a
slightly lower price that is substantially
above the variable costs of internal
production.
Answer (D) is incorrect because the
variable-cost-plus price is the price set
by charging for variable costs plus a
lump sum or an additional markup, but
less than full markup.
[19] Source: CIA 1190 IV-20
Answer (A) is incorrect because
inefficiencies are charged to the buying
department.
Answer (B) is correct. The optimal
transfer price of a selling division
should be set at a point that will have
the most desirable economic effect on
the firm as a whole while at the same
time continuing to motivate the
management of every division to
perform efficiently. Setting the transfer
price based on actual costs rather than
standard costs would give the selling
division little incentive to control costs.
Answer (C) is incorrect because, by
definition, cost-based transfer prices
are not adjusted by a markup or
markdown.
Answer (D) is incorrect because
cost-based transfer prices provide the
advantages of clarity and administrative
convenience.
[20] Source: CIA 1183 IV-5
Answer (A) is incorrect because
outside purchase will increase the
company's cost of sales by $10 per unit.
Answer (B) is incorrect because the

transfer price is irrelevant to the


decision. It does not affect overall
profits.
Answer (C) is incorrect because the
company is initially concerned with
covering variable rather than fixed
costs.
Answer (D) is correct. Opportunity
costs are $0 because A's facilities
would be idle if B did not purchase
from A. Assuming fixed costs are not
affected by the decision, the
intracompany sale is preferable from
the company's perspective because A's
$30 variable unit cost is less than the
outside vendor's price of $40.
[21] Source: CIA 0592 IV-19
Answer (A) is incorrect because $12
per unit merely allows Division Z to
recover its unit variable cost.
Answer (B) is correct. A unit price of
$18 is less than Division Y's cost of
purchase from an outside supplier but
exceeds Division Z's production cost.
Accordingly, both Y and Z benefit.

[23] Source: CIA 0588 IV-19


Answer (A) is incorrect because the
alpha division has no idle capacity.
Answer (B) is incorrect because the
opportunity cost needs to be included.
Answer (C) is incorrect because the
minimum transfer price equals outlay
(variable) costs plus opportunity cost,
not variable costs plus fixed costs.
Answer (D) is correct. In a perfectly
competitive market, market price is
ordinarily the appropriate transfer
price. Because the market price is
objective, using it avoids waste and
maximizes efficiency. In a perfectly
competitive market, the market price
equals the minimum transfer price,
which is the sum of outlay cost and
opportunity cost. Outlay cost is the
variable cost per unit, or $34 ($34,000
1,000). Opportunity cost is the
contribution margin foregone, or $16
($50 - $34). Thus, the minimum transfer
price is $50 ($34 + $16).
[24] Source: CIA 0593 IV-16

Answer (C) is incorrect because, at $20


per unit, Division Y may be indifferent
as to whether it purchases internally or
externally. Buying from an outside
source for $20 per unit is contrary to the
company's interests given idle capacity
available for the component's
manufacture and an incremental unit
cost of $20.

Answer (A) is incorrect because


internally developed information should
be developed whether or not transfer
prices are used.

Answer (D) is incorrect because, at $22


per unit, Division Y would have
incentive to purchase from an external
supplier (i.e. market price is $20).

Answer (C) is incorrect because


externally developed information is
needed for a market-based transfer
price.

[22] Source: CIA 0589 IV-16


Answer (A) is correct. From the seller's
perspective, the price should reflect at
least its incremental cash outflow
(outlay cost) plus the contribution from
an outside sale (opportunity cost).
Because A has idle capacity, the
opportunity cost is $0. Thus, the
minimum price Division A should
charge Division B is $7.00.
Answer (B) is incorrect because $7.00
is the minimum that should be charged.
Since Division A has idle capacity, the
minimum transfer price should recover
variable costs ($7.00).
Answer (C) is incorrect because
Division A should not include any fixed
costs in their transfer price because
Division A has idle capacity.
Answer (D) is incorrect because, since
Division A has idle capacity, the
minimum transfer price should recover
Division A's variable (outlay) costs.

Answer (B) is incorrect because market


effects on company operations are
characteristic of a market-based
transfer price.

Answer (D) is correct. A cost-based


transfer price is a price charged in an
intracompany transaction that covers
only the selling subunit's costs.
However, by ignoring relevant
alternative market prices, a company
may pay more than is necessary to
produce goods and services internally.
[25] Source: CIA 1193 IV-18
Answer (A) is incorrect because
marginal production cost transfer prices
do not relate to market-based transfer
prices.
Answer (B) is incorrect because
marginal cost based transfer prices
provide more of an incentive to the
purchasing division to buy internally
and thus use idle facilities of the selling
division than does the usually higher
market-based transfer price.
Answer (C) is correct. A transfer price
is the price charged in an intercompany
transaction. Market-based prices
provide market discipline because
efficient internal suppliers will tend to

prosper, thereby enhancing the overall


long-term competitiveness of the firm.
Answer (D) is incorrect because
corporate politics is less of a factor
than in other methods, such as a
negotiated transfer price. Market-based
prices are objective.

calculating transfer prices for a firm


may correctly price the firm's product in
Country A but not in Country B. The
product may be overpriced in Country
B, causing sales to be lower than
anticipated. Alternatively, the product
may be underpriced in Country B, and
the authorities may allege that the firm
is dumping its product there.

[26] Source: CIA 1193 IV-19


[28] Source: CIA 0594 III-71
Answer (A) is correct. A transfer price
is the price charged by one segment of
an organization for a product or service
supplied to another segment of the same
organization. The three basic criteria
that the transfer pricing system in a
decentralized company should satisfy
are to (1) provide information allowing
central management to evaluate
divisions with respect to total company
profit and each division's contribution
to profit, (2) stimulate each manager's
efficiency without losing each
division's autonomy, and (3) motivate
each divisional manager to achieve
his/her own profit goal in a manner
contributing to the company's success.
Because the $220 transfer price
selected is based on the quoted external
price (market), it is an example of
market-based transfer pricing.
Answer (B) is incorrect because the
cost-based price would be $210 ($100
+ $50 + $40 + $20).
Answer (C) is incorrect because no
negotiations took place.
Answer (D) is incorrect because cost
plus 20% would be $252 ($210 x 1.20).

Answer (A) is incorrect because


responsibility accounting holds
managers responsible only for what
they can control.
Answer (B) is incorrect because a cost
center manager is concerned with costs
only, whereas a profit center manager is
concerned with costs and revenues.
Answer (C) is correct. Responsibility
accounting stresses that managers are
responsible only for factors under their
control. For this purpose, the operations
of the business are organized into
responsibility centers. Costs are
classified as controllable and
uncontrollable. This implies that some
revenues and costs can be changed
through effective management.
Management may then focus on
deviations for either reinforcement or
correction. Thus, the statement that
every factor is ultimately controllable
by someone is not a premise of
responsibility accounting.
Answer (D) is incorrect because this is
the essence of responsibility
accounting. Each manager is held
accountable for factors under their
control.

[27] Source: CIA 0594 III-40


Answer (A) is incorrect because
properly chosen transfer prices allow
firms to attempt to minimize worldwide
taxes by producing various parts of the
products in different countries and
strategically transferring the parts at
various systematically calculated
prices.
Answer (B) is incorrect because
properly chosen transfer prices allocate
revenues and expenses to divisions in
various countries. These numbers are
used as part of the input for the
performance evaluation of each
division.
Answer (C) is incorrect because
transfer prices motivate division
managers to buy parts and products
(from either internal or external
suppliers) at the lowest possible prices
and to sell their products (to either
internal or external customers) at the
highest possible prices. Hence, each
division has a profit making orientation.
Answer (D) is correct. The calculation
of transfer prices in the international
arena must be systematic. A scheme for

[29] Source: CIA 1191 IV-18


Answer (A) is incorrect because
corporate administrative costs should
be excluded from the performance
report. The segments have no control
over their incurrence or the allocation
basis. The allocation depends upon the
segment sales (controllable) as well as
the sales of other segments
(uncontrollable).
Answer (B) is incorrect because the
segments have no control over the
incurrence of personnel costs or the
method of assignment, which depends
upon the number of employees in the
segment (controllable) in proportion to
the total number of employees in all
segments (not controllable).
Answer (C) is incorrect because the
segments have no control over fixed
computer facility costs, and the equal
assignment is arbitrary and bears no
relation to usage.
Answer (D) is correct. The variable
computer cost can be included. The
segments are charged for actual usage,

which is under each segment's control.


The predetermined standard rate is set
at the beginning of the year and is
known by the segment managers.
Moreover, the efficiencies and
inefficiencies of the computer
department are not passed on to the
segments. Both procedures promote a
degree of control by the segments.
[30] Source: CIA 1192 IV-22
Answer (A) is incorrect because
contribution margin reporting separates
costs by behavior; variable costs are
listed first followed by fixed costs.
Some responsibility accounting systems
use a contribution margin reporting
format, but contribution margin
reporting alone can include costs not
controllable by a manager.
Answer (B) is incorrect because
segment reporting is preparation of
performance reports by reportable
segments. Segment reports often include
allocated costs that are not controllable
by managers.
Answer (C) is incorrect because
absorption cost accounting is
characterized by its treatment of fixed
manufacturing overhead as a product
cost.
Answer (D) is correct. Responsibility
accounting stresses that managers
should only be held responsible for
factors under their control. To achieve
this objective, the operations of the
business are broken down into
responsibility centers. Costs are
classified as controllable and
noncontrollable to assign responsibility.
The assignment of responsibility
implies that some revenues and costs
can be changed through effective
management. A responsibility
accounting system should have certain
controls that provide for feedback
reports indicating deviations from
expectations. Management may then
focus on those deviations for either
reinforcement or correction.

revenues and expenses (s)he can


control. Controllability is the basic
concept of responsibility accounting.
[32] Source: Publisher
Answer (A) is correct. A responsibility
accounting system should have certain
controls that provide for feedback
reports indicating deviations from
expectations. Management may then
focus on those deviations (exceptions)
for either reinforcement or correction.
Answer (B) is incorrect because the
responsibility accounting system should
not be used exclusively to assess blame.
Answer (C) is incorrect because the
responsibility accounting system should
not be used exclusively to give
rewards.
Answer (D) is incorrect because
feedback reports concentrate on
deviations, but not to the total exclusion
of other budgeted variables.
[33] Source: Publisher
Answer (A) is incorrect because
generally accepted accounting
principles concern external financial
reporting, not internal reporting.
Answer (B) is incorrect because the
Financial Accounting Standards Board
concerns external financial reporting,
not internal reporting.
Answer (C) is incorrect because the
American Institute of Certified Public
Accountants concerns external financial
reporting, not internal reporting.
Answer (D) is correct. The
responsibility for internal reports is
management's. Management may direct
the accountant to provide a report in any
format deemed suitable for the decision
process. The accountant should work
closely with management to make these
reports an effective communication
device regarding the firm and its
decisions.

[31] Source: CIA 0589 IV-15


Answer (A) is incorrect because all
variable costs may not be controllable,
but some, if not all, fixed costs might be
controllable.
Answer (B) is incorrect because not all
budgeted costs are controllable by
managers.
Answer (C) is incorrect because all
product costs may not be controllable,
but some, if not all, period costs might
be controllable.
Answer (D) is correct. The most
desirable measure for evaluating a
departmental manager is one that holds
the manager responsible for the

[34] Source: CIA 1191 IV-17


Answer (A) is correct. The
responsibility accounting report should
list only the costs over which the
warehousing supervisor exercises
control. The supervisor's salary should
therefore be excluded because it is
controlled by the warehouse
supervisor's superior. Moreover, only
the product costs are to be considered.
These exclude the shipping clerks'
wages and fringe benefits because they
are period costs (shipping is a selling
expense). Thus, the only product cost
under the control of the warehouse
supervisor is the receiving clerks'
wages ($75,000) and the related fringe

benefits (.3 x $75,000 = $22,500), or a


total of $97,500.
[37] Source: Publisher
Answer (B) is incorrect because
$128,700 equals 130% of the sum of
60% of the supervisor's salary, and
100% of the receiving clerks wages.
Answer (C) is incorrect because
$130,000 is the sum of the wages of the
receiving and shipping clerks.
Answer (D) is incorrect because
$169,000 includes the shipping and
receiving clerks' wages and their
employee benefit costs. These should
be treated as period costs.
[35] Source: CIA 1190 IV-21
Answer (A) is correct. Residual income
is income of an investment center,
minus an imputed interest charge for
invested capital. The theory is that
earning an income greater than residual
income indicates that expansion is
desirable. However, comparisons of
investment centers based on residual
income may be misleading because of
differences in products, markets, costs,
and local conditions.
Answer (B) is incorrect because use of
the same imputed interest rate provides
a consistent objective against which
each investment can be measured.
Answer (C) is incorrect because
common amounts of invested capital
would eliminate a major factor causing
differences in residual income.
Answer (D) is incorrect because
comparisons of investment centers
based on residual income may be
misleading because of peculiarities of
each investment center (i.e. differences
in products, markets, costs, and local
conditions).

Answer (A) is incorrect because net


income as a percentage of sales is
calculated by dividing 6% (minimum
ROI) of the investment by the sales of
Segment B.
Answer (B) is incorrect because 6.67%
is the net income as a percentage of
sales for Segment C.
Answer (C) is correct. Residual income
was zero, indicating that net income
was equal to the minimum ROI. Given a
6% minimum ROI as a percentage of
investment, 6% of the $500,000
investment is $30,000. Sales were
$750,000, so net income ($30,000) is
4% of sales.
Answer (D) is incorrect because 10%
is the minimum ROI percentage for
Segment D.
[38] Source: Publisher
Answer (A) is incorrect because 5% is
the net income as a percentage of sales
for Segment D.
Answer (B) is correct. Net income as a
percentage of sales is the ROI divided
by turnover of investment. The turnover
of the investment is sales ($135,000)
divided by the investment ($45,000), or
3. Hence, net income is 6.67% (20%
3) as a percentage of sales for Segment
C.
Answer (C) is incorrect because 4% is
the net income as a percentage of sales
for Segment B.
Answer (D) is incorrect because 20%
is the ROI for Segment C.
[39] Source: Publisher

[36] Source: Publisher


Answer (A) is incorrect because
motivation is the desire and the
commitment to achieve a specific goal.
Answer (B) is incorrect because
autonomy is the extent to which
individuals have the authority to make
decisions.
Answer (C) is correct. Managerial
effort is the extent to which a manager
attempts to accomplish a goal.
Managerial effort may include
psychological as well as physical
commitment to a goal.
Answer (D) is incorrect because
managerial effort reflects the attempts
of an individual manager or
organizational unit, not those efforts
shared among managers of different
organization units.

Answer (A) is correct. The turnover of


investment for Segment C is calculated
by dividing sales by investment. Given
sales of $135,000 and investment of
$45,000, Segment C's turnover of
investment is 3.
Answer (B) is incorrect because 1.5 is
the turnover of investment for Segment
B.
Answer (C) is incorrect because 2.5 is
the turnover of investment for Segment
A.
Answer (D) is incorrect because the
turnover of investment is calculated by
dividing sales by investment.
[40] Source: Publisher
Answer (A) is correct. The turnover of

investment for Segment D is determined


by dividing sales by investment. For
Segment D, net income ($90,000) as a
percentage of sales ($1,800,000) equals
5%. ROI is given as 7.5%. Dividing net
income as a percentage of sales (5%)
into ROI (7.5%) gives a turnover of
investment of 1.5.

for Segment D.
Answer (D) is incorrect because
$12,000 results from multiplying sales
(rather than investment) by the minimum
rate of return percentage.
[44] Source: Publisher

Answer (B) is incorrect because 2.5 is


the turnover of investment for Segment
A.
Answer (C) is incorrect because the
turnover of investment is calculated by
dividing sales by investment.
Answer (D) is incorrect because 20
results from dividing sales by net
income.

Answer (A) is correct. The ROI in


dollars is equal to the amount of the
investment times the minimum rate of
return percentage. The amount of the
investment is $500,000. The minimum
rate of return percentage is given as
6%. Thus, the minimum ROI in dollars
equals $30,000.
Answer (B) is incorrect because $6,750
is the minimum dollar ROI for Segment
C.

[41] Source: Publisher


Answer (A) is incorrect because ROI is
calculated by dividing net income by
investment.
Answer (B) is correct. ROI is equal to
net income divided by investment. Net
income equals $5,000. Investment
equals $24,000. ROI equals 20.8%.
Answer (C) is incorrect because ROI is
calculated by dividing net income by
investment.
Answer (D) is incorrect because 8.3%
is the net income as a percent of sales
(5,000/60,000).
[42] Source: Publisher
Answer (A) is correct. Residual income
is given as zero. Thus, the actual ROI is
the same as the minimum percentage
ROI of 6%.
Answer (B) is incorrect because ROI is
calculated by dividing net income by
investment.
Answer (C) is incorrect because 20%
is the ROI for Segment C.
Answer (D) is incorrect because 7.5%
is the ROI for Segment D.

Answer (C) is incorrect because $4,800


is the minimum dollar ROI for Segment
A.
Answer (D) is incorrect because
$45,000 results from multiplying sales
(rather than investment) by the minimum
rate of return percentage.
[45] Source: Publisher
Answer (A) is correct. The minimum
ROI in dollars is equal to the minimum
ROI percentage times the investment.
The investment was $45,000. Neither
the minimum percentage nor the
minimum ROI is known. However, the
ROI percentage (20%) and the
investment ($45,000) are known.
Hence, the net income is $9,000. Given
residual income of $2,250, the minimum
ROI in dollars must have been $6,750
($9,000 - $2,250).
Answer (B) is incorrect because $4,800
is the minimum dollar ROI for Segment
A.
Answer (C) is incorrect because
$120,000 is the minimum dollar ROI
for Segment D.
Answer (D) is incorrect because
$9,000 results form multiplying
investment by actual (rather than
minimum) ROI.

[43] Source: Publisher


Answer (A) is incorrect because
$30,000 is the minimum dollar ROI for
Segment B.
Answer (B) is correct. The ROI in
dollars is equal to the amount of the
investment times the minimum rate of
return percentage. The amount of the
investment is $24,000. The minimum
rate of return percentage is 20%.
Accordingly, the minimum ROI in
dollars is $4,800.
Answer (C) is incorrect because
$120,000 is the minimum dollar ROI

[46] Source: Publisher


Answer (A) is incorrect because 20%
is the minimum ROI percentage for
Segment A.
Answer (B) is incorrect because 6% is
the minimum ROI percentage for
Segment B.
Answer (C) is correct. The minimum
percentage of ROI in Segment C equals
the minimum dollar ROI divided by the
investment. The minimum dollar ROI is
$6,750. Consequently the minimum

percentage ROI is 15% ($6,750


$45,000).
Answer (D) is incorrect because 10%
is the minimum ROI percentage for
Segment D.
[47] Source: Publisher
Answer (A) is incorrect because 20%
is the minimum ROI percentage for
Segment A.
Answer (B) is incorrect because 6% is
the minimum ROI percentage for
Segment B.
Answer (C) is incorrect because 15%
is the minimum ROI percentage for
Segment C.
Answer (D) is correct. The minimum
percentage of ROI for Segment D is the
minimum ROI in dollars ($120,000)
divided by the investment, which must
be calculated. The ROI is given as
7.5%. The net income ($90,000) as a
percentage of sales ($1,800,000) equals
5%. The turnover of investment (ROI
net income as a percentage of sales) is
1.5. Given turnover of 1.5 and sales of
$1,800,000, investment must have been
$1,200,000. The minimum percentage
ROI is $120,000 divided by the
$1,200,000 investment, or 10%.
[48] Source: Publisher
Answer (A) is correct. Segment A's
residual income is equal to the net
income ($5,000) minus the minimum
ROI in dollars. Minimum ROI in
dollars equals the minimum ROI
percentage (20%) times the investment
($24,000), or $4,800. Residual income
is therefore $200.
Answer (B) is incorrect because
$12,000 is the minimum ROI percentage
multiplied by sales.
Answer (C) is incorrect because
$(30,000) is the residual income for
Segment D.
Answer (D) is incorrect because the
residual income for Segment A is
determined by subtracting the minimum
ROI dollars from net income.
[49] Source: Publisher
Answer (A) is incorrect because the
residual income for Segment D is
determined by subtracting the minimum
ROI dollars from net income.
Answer (B) is correct. The minimum
ROI in dollars is given as $120,000 and
net income is given as $90,000. Thus,
residual income is $(30,000) ($90,000
- $120,000). Segment D did not achieve
its minimum ROI and therefore has a
negative residual income.

Answer (C) is incorrect because the


residual income for Segment D is
determined by subtracting the minimum
ROI dollars from net income.
Answer (D) is incorrect because the
residual income for Segment D is
determined by subtracting the minimum
ROI dollars from net income.
[50] Source: Publisher
Answer (A) is incorrect because fixed
costs per unit decrease because there
are more units to absorb the fixed costs
and variable costs do not change over
the relevant range.
Answer (B) is incorrect because fixed
costs per unit decrease because there
are more units to absorb the fixed costs.
Answer (C) is incorrect because
variable costs per unit do not change
over the relevant range.
Answer (D) is correct. Fixed costs per
unit decrease within the relevant range
of activity as production increases
because more units are available to
absorb the constant amount of total
fixed costs. Unit variable costs are
assumed to remain the same per unit
over the relevant range.
[51] Source: Publisher
Answer (A) is incorrect because a cost
center is not responsible for revenues.
Answer (B) is incorrect because the
performance reports of an investment
center and a profit center but not a cost
center include controllable revenues.
Answer (C) is correct. In investment
centers, managers are responsible for
all activities, including costs, revenues,
and investments. An investment center
is a profit center with significant
control over the amount of capital
invested. This control extends to
investments such as receivables and
property, plant, and equipment, as well
as entry into new markets. A cost
center, for example, a production
department, is responsible for costs
only. A profit center, for example, the
appliance department in a retail store,
is responsible for both revenues and
expenses.
Answer (D) is incorrect because
investment centers and profit centers
are responsible for revenues.
[52] Source: Publisher
Answer (A) is correct. Allocated fixed
overhead should not be included in
internal reports based on a
responsibility accounting system
because it cannot be controlled by a

manager of a responsibility center.


Answer (B) is incorrect because, in
responsibility accounting, managers are
only held responsible for costs they
have the authority to control.
Answer (C) is incorrect because the
organizational chart, which outlines the
authority-responsibility chain of a
company, is an integral part of the
responsibility accounting system.
Answer (D) is incorrect because a main
purpose of internal reports is to show
the variance between actual and
budgeted controllable costs so
corrective action can be taken when and
where needed.
[53] Source: Publisher
Answer (A) is correct. A responsibility
center is a subunit of an organization
whose manager is accountable for a
specified set of activities. Job-order
costing, process costing, and
activity-based costing all may
accumulate their costs by responsibility
centers.

Answer (D) is incorrect because the


profit center's manager does not control
the listed period expenses and therefore
does not control the profit center's
income.
[55] Source: Publisher
Answer (A) is incorrect because
suboptimization occurs when a decision
benefits a segment of a company but not
the company as a whole.
Answer (B) is incorrect because
suboptimization occurs when a decision
benefits a segment of a company but not
the company as a whole.
Answer (C) is incorrect because
suboptimization occurs when a decision
benefits a segment of a company but not
the company as a whole.
Answer (D) is correct. Suboptimization
occurs when one segment of a company
takes an action that benefits itself but
not the firm as a whole.
[56] Source: Publisher

Answer (B) is incorrect because


job-order costing may also accumulate
costs by responsibility centers for
control purposes.
Answer (C) is incorrect because
process costing and activity-based
costing may also accumulate their costs
by responsibility centers for control
purposes.
Answer (D) is incorrect because
job-order costing, process costing, and
activity-based costing may accumulate
their costs by responsibility centers.

Answer (A) is incorrect because the


ROI equals capital turnover (sales
divided by investment) times the profit
margin (net income divided by sales).
Answer (B) is incorrect because the
ROI equals capital turnover (sales
divided by investment) times the profit
margin (net income divided by sales).
Answer (C) is correct. ROI equals
capital turnover (sales divided by
investment) times the profit margin (net
income divided by sales). Therefore,
Andrew's ROI is 25% [($720,000
$480,000) x ($120,000 $720,000)].

[54] Source: Publisher


Answer (A) is correct. A profit center
is a segment of a company responsible
for both revenues and expenses. A
profit center has the authority to make
decisions concerning markets
(revenues) and sources of supplies
(costs). However, the profit center's
manager does not control his/her salary,
investment and the resulting costs (e.g.,
depreciation of plant assets), or
expenses incurred at the corporate
level. Consequently, profit center No.
12 is most likely to control the $84,000
contribution margin (sales - variable
costs) but not the other items in the
summarized income statement.
Answer (B) is incorrect because the
profit center manager does not control
depreciation on accommodations
($9,600) or the allocated corporate
expenses ($6,000).
Answer (C) is incorrect because the
profit center manager does not control
his/her $24,000 salary.

Answer (D) is incorrect because the


ROI equals capital turnover (sales
divided by investment) times the profit
margin (net income divided by sales).
[57] Source: Publisher
Answer (A) is correct. Residual income
is income of an investment center minus
an imputed interest charge for invested
capital. Accordingly, Cinder's residual
income is $170,400 [($600,000 sales $360,000 variable costs - $60,000
traceable fixed costs) net income - (8%
x $120,000 average invested capital)
imputed interest].
Answer (B) is incorrect because the
imputed interest charge of $9,600 (8% x
$120,000) must be deducted.
Answer (C) is incorrect because the
imputed interest charge of $9,600
should be subtracted from, not added to,
net income.

Answer (D) is incorrect because the


traceable fixed costs must be deducted.

[60] Source: Publisher


Answer (A) is correct. The breakeven
point in units is total fixed costs divided
by the unit contribution margin (UCM).
The UCM is the selling price minus
variable costs per unit. Variable costs

[58] Source: Publisher


Answer (A) is incorrect because 18%
is the rate at which Ash imputes interest
on investments.

per unit equal $1.50 ($540,000


360,000 units). Thus, the UCM equals
$.90 ($2.40 - $1.50). Dividing the
$108,000 of fixed costs by the $.90
UCM yields a breakeven point of
120,000 units.

Answer (B) is incorrect because 25.7%


results from using year 2 year-end
invested capital instead of average
invested capital.
Answer (C) is correct. The return on
average investment equals net income
from operations divided by average
invested capital. Average invested
capital is $720,000 [($600,000 +
$840,000) 2].
Sales (360,000 units x $2.40)
Minus:
Variable costs
Contribution margin
Minus:
Fixed costs

Answer (B) is incorrect because selling


110,000 units would result in a loss of
$.08 per unit.
Answer (C) is incorrect because selling
100,000 units would result in a loss of
$.18 per unit.
$864,000

(540,000)
????????
$324,000

(108,000)
????????
Net operating income
$216,000
========
Therefore, the return on average
investment was 30% ($216,000
$720,000).
Answer (D) is incorrect because 36%
results from using year 1 year-end
invested capital instead of average
invested capital.

Answer (D) is incorrect because selling


90,000 units would result in a loss of
$.30 per unit.
[61] Source: Publisher
Answer (A) is incorrect because
contribution margin is sales minus total
variable costs.
Answer (B) is correct. The contribution
margin is sales of $864,000 (360,000
units x $2.40) minus variable costs of
$540,000, or $324,000.
Answer (C) is incorrect because
contribution margin is sales minus total
variable costs.

[59] Source: Publisher


Answer (A) is correct. Residual income
is equal to net operating income minus
imputed interest on invested capital.
Net operating income and average
investment were $72,000 and $720,000,
respectively. The imputed interest rate
is 18%. Thus,
Net operating income
$ 72,000
Minus:
Imputed interest (18% x $720,000)
(129,600)
?????????
Residual income
$ (57,600)
=========
Answer (B) is incorrect because the
imputed interest is 18% of the average
invested capital of $720,000, not of the
year-end invested capital of $840,000.
Answer (C) is incorrect because the
imputed interest of $129,600 (18% x
$720,000) needs to be deducted from
net operating income.
Answer (D) is incorrect because the
imputed interest is 18% of the average
invested capital of $720,000, not of the
capital at the beginning of the year of
$600,000.

Answer (D) is incorrect because


contribution margin is sales minus total
variable costs.
[62] Source: Publisher
Answer (A) is incorrect because
$648,000 is the total costs and expenses
for year 2 based on a production of
360,000 hammers.
Answer (B) is correct. Over the
relevant range, fixed costs will not
fluctuate. The variable cost per unit was
$1.50. Thus, total costs and expenses
will be
Variable ($1.50 x 400,000 units)
$600,000
Fixed
108,000
????????
Total costs and expenses
$708,000
========
Answer (C) is incorrect because, within
the relevant range, fixed costs will
remain constant at $108,000.
Answer (D) is incorrect because,
within the relevant range, fixed costs
will remain constant at $108,000.
[63] Source: CIA 0594 III-44

Answer (A) is incorrect because


$370,000 measures the performance of
the marketing segment manager
($950,000 - $430,000 - $150,000).
Answer (B) is incorrect because
$10,000 includes the allocated costs
($950,000 - $430,000 - $150,000 $250,000 - $110,000).
Answer (C) is incorrect because
$520,000 is the contribution margin
($950,000 - $430,000).
Answer (D) is correct. The best
measure of the segment's economic
performance includes all costs except
the fixed manufacturing costs allocated
to the segment. Thus, the best measure
of economic performance is $120,000
($950,000 - $430,000 - $150,000 $250,000).
[64] Source: CMA 1291 3-10
Answer (A) is incorrect because a
service center has no responsibility for
developing markets or selling.
Answer (B) is incorrect because a
production center is engaged in
manufacturing.
Answer (C) is incorrect because a
profit center can choose its markets and
sources of supply.
Answer (D) is correct. A service center
exists primarily and sometimes solely
to provide specialized support to other
units within the organization. Service
centers are usually operated as cost
centers.
[65] Source: CMA 0693 3-14
Answer (A) is incorrect because a
profit center is a segment responsible
for both revenues and costs. A profit
center has the authority to make
decisions concerning markets and
sources of supply.
Answer (B) is incorrect because an
investment center is a responsibility
center that is accountable for revenues
(markets), costs (sources of supply),
and invested capital.
Answer (C) is incorrect because a
contribution center is responsible for
revenues and variable costs, but not
invested capital.
Answer (D) is correct. A cost center is
a responsibility center that is
accountable only for costs. The cost
center is the least complex type of
segment because it has no responsibility
for revenues or investments.

Answer (A) is incorrect because


flexible budgeting is the process of
preparing a series of multiple budgets
for varying levels of production or
sales.
Answer (B) is incorrect because human
resource management is the process of
managing personnel.
Answer (C) is correct. Managerial
performance should ideally be
evaluated only on the basis of those
factors controllable by the manager.
Managers may control revenues, costs,
or investments in resources. A
well-designed responsibility accounting
system establishes responsibility
centers within the organization.
However, controllability is not an
absolute basis for establishment of
responsibility. More than one manager
may be able to influence a cost, and
responsibility may be assigned on the
basis of knowledge about the incurrence
of a cost rather than the ability to
control it. Management by objectives
(MBO) is a related concept. It is a
behavioral, communications-oriented,
responsibility approach to employee
self-direction. Under MBO, a manager
and his/her subordinates agree upon
objectives and the means of attaining
them. The plans that result are reflected
in responsibility accounting and in the
budgeting process.
Answer (D) is incorrect because capital
budgeting is a means of evaluating
long-term investments.
[67] Source: CMA 1293 3-21
Answer (A) is incorrect because fixed
costs may also be controllable, and
some costs not controllable may need to
be assigned.
Answer (B) is incorrect because
knowledge about the incurrence of a
cost rather than controllability may in
practice be an appropriate basis for
delegation of responsibility.
Answer (C) is correct. Managerial
performance should ideally be
evaluated only on the basis of those
factors controllable by the manager.
Managers may control revenues, costs,
and/or investments in resources.
However, controllability is not an
absolute. More than one manager may
be able to influence a cost, and
managers may be accountable for some
costs they do not control. In practice,
given the difficulties of determining the
locus of controllability, responsibility
may be assigned on the basis of
knowledge about the incurrence of a
cost rather than the ability to control it.
Accordingly, a successful system is
dependent upon the proper delegation of
responsibility and the commensurate
authority.

[66] Source: CMA 1293 3-17


Answer (D) is incorrect because fixed

costs can be controllable.

[68] Source: CMA 0694 3-24


Answer (A) is incorrect because a
revenue center is responsible only for
revenue generation, not for costs or
capital investment.
Answer (B) is incorrect because a
profit center is responsible for revenues
and costs but not for invested capital.

accounting holds managers responsible


only for factors under their control. For
this purpose, operations are organized
into responsibility centers. Costs are
classified as controllable and
noncontrollable, which implies that
some revenues and costs can be
changed through effective management.
If a manager has authority to incur costs,
a responsibility accounting system will
charge those costs to the manager's
responsibility center.
[71] Source: CMA 0695 3-28

Answer (C) is incorrect because a cost


center is evaluated only on the basis of
costs incurred. It is not responsible for
revenues or invested capital.
Answer (D) is correct. An investment
center is the organizational type most
like an independent business because it
is responsible for its own revenues,
costs incurred, and capital invested.
The other types of centers do not
incorporate all three elements.
[69] Source: CMA 1294 3-20
Answer (A) is correct. Responsibility
accounting holds managers responsible
only for factors under their control. For
this purpose, operations are organized
into responsibility centers. Costs are
classified as controllable and
noncontrollable, which implies that
some revenues and costs can be
changed through effective management.
If a manager has authority to incur costs,
a responsibility accounting system will
charge them to the manager's
responsibility center.
Answer (B) is incorrect because
functional accounting allocates costs to
functions regardless of responsibility.
Answer (C) is incorrect because
reciprocal allocation is a means of
allocating service department costs.
Answer (D) is incorrect because
transfer price accounting is a means of
charging one department for products
acquired from another department in the
same organization.
[70] Source: CMA 1294 3-21
Answer (A) is incorrect because
contribution accounting is a method of
control in which only variable costs are
matched with revenues.
Answer (B) is incorrect because
cost-benefit accounting is a nonsense
term referring to matching costs and
benefits.
Answer (C) is incorrect because
flexible budgeting prepares budgets for
multiple levels of operations.
Answer (D) is correct. Responsibility

Answer (A) is incorrect because many


overhead costs are also controllable.
Answer (B) is correct. Control is the
process of making certain that plans are
achieving the desired objectives. A
controllable cost is one that is directly
regulated by a specific manager at a
given level of production within a given
time span. For example, fixed costs are
often not controllable in the short run.
Answer (C) is incorrect because
controllable costs need not be
discretionary. Discretionary costs are
characterized by uncertainty about the
relationship between input and the
value of the related output; they may or
may not be controllable.
Answer (D) is incorrect because
controllable costs are those over which
a manager has control; the manager may
be knowledgeable and informed about
costs that (s)he cannot control.
[72] Source: CMA 1295 3-5
Answer (A) is correct. A profit center
is responsible for both revenues and
costs, whereas a cost center is
responsible only for costs.
Answer (B) is incorrect because a
revenue center is responsible only for
revenues, not costs.
Answer (C) is incorrect because a
division can be any type of
responsibility center.
Answer (D) is incorrect because an
operating unit can be organized as any
type of center.
[73] Source: CMA 1296 3-16
Answer (A) is incorrect because direct
labor is controllable by the production
manager.
Answer (B) is incorrect because
materials are controllable by the
production manager.
Answer (C) is incorrect because
repairs and maintenance are
controllable by the production manager.

Answer (D) is correct. A well-designed


responsibility accounting system
establishes responsibility centers within
an organization. In a responsibility
accounting system, managerial
performance should be evaluated only
on the basis of those factors
controllable (or at least capable of
being significantly influenced) by the
manager. Thus, a manager of an
assembly line is responsible for direct
labor, materials, repairs and
maintenance, and supervisory salaries.
The manager is not responsible for
depreciation on the manufacturing
facility. (S)he is not in a position to
control or influence capital budgeting
decisions.
[74] Source: CMA 0692 3-23
Answer (A) is incorrect because
nothing indicates that a flexible budget
is used.
Answer (B) is correct. Responsibility
accounting stresses that managers
should only be held responsible for
factors under their control. To achieve
this objective, the operations of the
business are broken down into
responsibility centers. Costs are
classified as controllable and
noncontrollable to assign responsibility.
The assignment of responsibility
implies that some revenues and costs
can be changed through effective
management. A responsibility
accounting system should have certain
controls that provide for feedback
reports indicating deviations from
expectations. Management may then
focus on those deviations for either
reinforcement or correction.
Answer (C) is incorrect because the
cost-benefit constraint on accounting
information is pervasive. The benefits
of the information provided by any
system should at least equal the cost.
Answer (D) is incorrect because
program budgeting is a system in which
budgets are prepared by program rather
than by line item costs (such as salaries
or supplies). It is a long-term method
that associates outlays with the broad
objectives of the organization.

that provide for feedback reports


indicating deviations from expectations.
Higher-level management may focus on
those deviations for either
reinforcement or correction.
Answer (C) is incorrect because
authority is an element of a
responsibility accounting system, not
the basic purpose.
Answer (D) is incorrect because
analysis of variances is an element of a
responsibility accounting system, not
the basic purpose.
[76] Source: CMA 0693 3-29
Answer (A) is incorrect because
allocating depreciation on the basis of
long-term average use is a reasonable
basis of allocation. This basis is
controllable by the division managers
and reflects a causal relationship.
Answer (B) is correct. Managerial
performance ordinarily should be
evaluated only on the basis of those
factors controllable by the manager. If a
manager is allocated costs that (s)he
cannot control, dysfunctional motivation
can result. In the case of allocations, a
cause-and-effect basis should be used.
Allocating the costs of upkeep on a
headquarters building on the basis of
sales revenue is arbitrary because cost
may have no relationship to divisional
sales revenues. Consequently,
divisional ROI is reduced by a cost
over which a division manager has no
control. Furthermore, the divisions with
the greatest sales are penalized by
receiving the greatest allocation.
Answer (C) is incorrect because a
service department's cost overruns may
not be attributable to any activities of
production departments.
Answer (D) is incorrect because
market-based allocations of costs of
services are reasonable applications of
the cause-and-effect principle.
[77] Source: CMA 0686 4-14
Answer (A) is incorrect because sales
of the division would appear on the
statement.

[75] Source: CMA 0691 3-28


Answer (A) is incorrect because
budgeting is an element of a
responsibility accounting system, not
the basic purpose.
Answer (B) is correct. The basic
purpose of a responsibility accounting
system is to motivate management to
perform in a manner consistent with
overall company objectives. The
assignment of responsibility implies
that some revenues and costs can be
changed through effective management.
The system should have certain controls

Answer (B) is incorrect because the


division's fixed selling expenses are
separable fixed costs.
Answer (C) is incorrect because
variable costs of the division are
included.
Answer (D) is correct. As defined in
Statement on Management Accounting
2A, Management Accounting Glossary,
segment margin is the contribution
margin for a segment of a business
minus fixed costs. It is a measure of

long-run profitability. Thus, an


allocation of the corporate officers'
salaries should not be included in
segment margin because they are neither
variable costs nor fixed costs that can
be rationally allocated to the segment.
Other items that are often not allocated
include corporate income taxes,
interest, company-wide R&D expenses,
and central administration costs.

are classified as controllable and


noncontrollable to assign responsibility,
which implies that some revenues and
costs can be changed through effective
management. For example, depreciation
on equipment is ordinarily not
controllable by the manager of an
assembly line and should not appear on
his/her performance report.
[80] Source: CMA 1292 3-22

[78] Source: CIA 0587 IV-15


Answer (A) is correct. Managerial
performance should be evaluated only
on the basis of those factors
controllable by the manager. Managers
may control revenues, costs, and
investment in resources. A
well-designed responsibility accounting
system establishes responsibility
centers within the organization. The
sales department should therefore be
responsible for the overtime costs
because it can best judge whether the
additional cost of the rush order is
justified. Therefore, the sales
department may be charged with the
overtime costs because it can best judge
whether the additional cost of the rush
order is justified. The production
department may also be held
responsible for the overtime costs
because charging the full overtime cost
to the sales department would give the
production department no incentive to
control these costs. However, the
personnel department would never be
charged with the overtime costs
because it has no effect on the
incurrence of production overtime.
Answer (B) is incorrect because, to
control costs the production department
may be charged with the overtime costs.

Answer (A) is incorrect because


unallocated fixed costs do not affect
either performance measure.
Answer (B) is incorrect because direct
variable costs affect both performance
measures.
Answer (C) is incorrect because costs
controllable by the manager affect both
performance measures.
Answer (D) is correct. Control of costs
accounts for the major difference
between segment manager performance
and segment performance. Segment
performance is based on all costs
directly attributable to the segment.
Segment manager performance is based
on all costs directly controllable by the
segment manager. All variable costs
ordinarily meet the criteria for both
measures. The difference usually arises
because a fixed cost is directly
attributable to a segment but is not
controllable by the manager. For
example, a profit center manager may
have no control over fixed costs of the
segment.
[81] Source: CMA 0691 3-29

Answer (C) is incorrect because, to


control costs, the sales department may
be charged with the overtime costs.

Answer (A) is incorrect because total


assets available does not reflect
financing decisions.

Answer (D) is incorrect because, to


control costs, the sales department and
the production department may be
charged with the overtime costs.

Answer (B) is incorrect because total


assets employed does not reflect
financing decisions.

[79] Source: CMA 0691 3-26


Answer (A) is incorrect because
supervisory salaries are costs
controllable by an assembly line
manager.
Answer (B) is incorrect because
materials are costs controllable by an
assembly line manager.
Answer (C) is incorrect because
repairs and maintenance are costs
controllable by an assembly line
manager.
Answer (D) is correct. Responsibility
accounting stresses that managers
should be held responsible for only
those factors under their control. Costs

Answer (C) is incorrect because


deducting current liabilities from
current assets to arrive at working
capital reflects the control that the
manager of the responsibility center
ordinarily has over short-term credit
transactions.
Answer (D) is correct. Stockholders'
equity equals total assets minus total
liabilities. The latter include short-term
liabilities incurred at operating levels
of the organization and long-term
liabilities resulting from financing
decisions made by top management.
Accordingly, the investment base used
to measure the performance of a
manager may reflect the incurrence of
liabilities over which (s)he had no
control. A second problem is that the
allocation of long-term liabilities
among divisions or segments may be

somewhat arbitrary.
[82] Source: CMA 1291 3-7
Answer (A) is incorrect because fixed
operating assets are controlled by the
division manager and contribute to
profits.
Answer (B) is correct. An evaluation of
an investment center is based upon the
return on the investment base. These
assets include plant and equipment,
inventories, and receivables. Most
likely, however, an asset, such as land,
that is being held by the division as a
site for a new plant would not be
included in the investment base because
it is not currently being used in
operations. Total assets in use rather
than total assets available is preferable
when the investment center has been
forced to carry idle assets.
Answer (C) is incorrect because
inventories are operating assets that
contribute to profits and are controlled
by the division manager.
Answer (D) is incorrect because the
level of accounts payable is an
operating decision that should be
considered in the evaluation of the
division manager.
[83] Source: CMA 1292 3-21
Answer (A) is incorrect because
increasing sales and expenses by the
same dollar amount will not change
income or ROI.
Answer (B) is incorrect because
decreasing revenues and expenses by
the same percentage will reduce income
and lower ROI.
Answer (C) is incorrect because
increasing investment and operating
expenses by the same dollar amount
will lower ROI. The higher investment
increases the denominator, and the
increased expenses reduce the
numerator.
Answer (D) is correct. ROI equals
income divided by invested capital. If a
company is already profitable,
increasing sales and expenses by the
same percentage will increase ROI. For
example, if a company has sales of
$100 and expenses of $80, its net
income is $20. Given invested capital
of $100, ROI is 20% ($20 $100). If
sales and expenses both increase 10%
to $110 and $88, respectively, net
income increases to $22. ROI will then
be 22% ($22 $100).
[84] Source: CMA 0693 3-12
Answer (A) is incorrect because
Sanders and Carolina would accept the
project.

Answer (B) is incorrect because


Sanders and Carolina would accept the
project.
Answer (C) is correct. Residual income
is the excess of the return on an
investment over a targeted amount,
which is equal to an imputed interest
charge on invested capital (in this case,
8%). The rate is usually the
weighted-average cost of capital. Some
enterprises prefer to measure
managerial performance in terms of the
amount of residual income rather than
the percentage ROI. The principle is
that the enterprise is expected to benefit
from expansion as long as residual
income is earned. Using a percentage
ROI approach, expansion might be
rejected if it lowered ROI, even though
residual income would increase. Using
residual income, both Carolina and
Sanders would accept the new project
because residual income will increase
if a 12% return is earned when the
target ROI is only 8%.
Answer (D) is incorrect because
Sanders and Carolina would accept the
project.
[85] Source: CMA 0693 3-11
Answer (A) is correct. A company with
an 8% ROI threshold should obviously
accept a project yielding 12% because
the company's overall ROI would
increase. The manager being evaluated
on the basis of ROI who is already
earning 14% will be unwilling to
accept a 12% return on a new project
because the overall ROI for the division
would decline slightly. This absence of
goal congruence suggests a weakness in
ROI-based performance evaluation.
Answer (B) is incorrect because
Carolina would accept a project
yielding a return greater than 8%, and
Sanders would reject a return yielding
less than 14%.
Answer (C) is incorrect because
Carolina would accept a project
yielding a return greater than 8%, and
Sanders would reject a return yielding
less than 14%.
Answer (D) is incorrect because
Carolina would accept a project
yielding a return greater than 8%, and
Sanders would reject a return yielding
less than 14%.
[86] Source: CMA 0693 3-27
Answer (A) is incorrect because ROI
can be misleading when the quality of
the investment base differs among
segments.
Answer (B) is correct. Return on
investment is the key performance
measure in an investment center. ROI is

a rate computed by dividing a segment's


income by the invested capital. ROI is
therefore subject to the numerous
possible manipulations of the income
and investment amounts. For example, a
manager may choose not to invest in a
project that will yield less than the
desired rate of return, or (s)he may
defer necessary expenses.
Answer (C) is incorrect because
managers may reject projects that are
profitable (a return greater than the cost
of capital), but would decrease ROI.
For example, a segment with a 15%
ROI may not want to invest in a new
project with a 10% ROI, even though
the cost of capital might be only 8%.
Answer (D) is incorrect because the use
of ROI does not reflect the relative
difficulty of tasks undertaken by
managers.
[87] Source: CMA 0694 3-18
Answer (A) is incorrect because the
cost of equity capital must also be
incorporated into the imputed interest
rate.
Answer (B) is incorrect because the
current weighted-average cost of
capital must be used.
Answer (C) is correct. Residual income
is the excess of the return on an
investment over a targeted amount equal
to an imputed interest charge on
invested capital. The rate used is
ordinarily set as a target return by
management but is often equal to the
weighted average cost of capital. Some
enterprises prefer to measure
managerial performance in terms of the
amount of residual income rather than
the percentage ROI because the firm
will benefit from expansion as long as
residual income is earned.
Answer (D) is incorrect because the
rate should be based on cost of capital,
not investment returns of preceding
years.
[88] Source: CMA 0694 3-29
Answer (A) is incorrect because ROI is
based on all assets, not just current
investment expenditures.
Answer (B) is incorrect because the
calculation of ROI does not adjust for
imputed interest on invested capital.
Answer (C) is incorrect because the
denominator would not be limited to
fixed assets.
Answer (D) is correct. ROI is
calculated by dividing income by
invested capital. It is a key performance
measure of an investment center.
Invested capital may be defined in
various ways, such as shareholders'

equity, total assets available, or total


assets employed (which excludes assets
that are idle). Total assets available is
the measure that assumes the manager
will use all assets without regard to
financing.
[89] Source: CMA 0695 3-20
Answer (A) is incorrect because,
although the firm's return on equity
investment was 4%, its return on all
funds invested was 5% ($25,000 pretax
operating income $500,000).
Answer (B) is correct. Return on
investment is commonly calculated by
dividing pretax income by total assets
available. Residual income is the
excess of the return on investment over
a targeted amount equal to an imputed
interest charge on invested capital. The
rate used is ordinarily the
weighted-average cost of capital. Some
companies measure managerial
performance in terms of the amount of
residual income rather than the
percentage return on investment.
Because REB has assets of $500,000
and a cost of capital of 6%, it must earn
$30,000 on those assets to cover the
cost of capital. Given that operating
income was only $25,000, it had a
negative residual income of $5,000.
Answer (C) is incorrect because ROI is
commonly based on before-tax income.
Answer (D) is incorrect because
$(22,000) equals the difference
between net profit after taxes and
targeted income.
[90] Source: CMA 1296 3-2
Answer (A) is incorrect because these
ROI computations do not subtract
imputed interest on capital used from
the investment base.
Answer (B) is correct. Residual income
is the excess of the amount of return on
investment (ROI) over a targeted
amount equal to an imputed interest
charge on invested capital. The rate
used to impute the interest is usually the
weighted-average cost of capital. The
advantage of using residual income
rather than percentage ROI is that the
former emphasizes maximizing an
amount instead of a percentage.
Managers are encouraged to accept
projects with returns exceeding the cost
of capital even if the investments reduce
the percentage ROI.
Answer (C) is incorrect because
operating income equals operating
revenues minus operating costs.
Answer (D) is incorrect because these
ROI computations do not subtract
imputed interest on capital used from
the investment base.

[91] Source: CMA 1296 3-27


Answer (A) is incorrect because they
reflect an assumption that the subunit
manager does not influence the resource
base (denominator of the ROI
calculation).
Answer (B) is correct. ROI equals
income divided by invested capital. The
denominator may be defined in various
ways, e.g., total assets available, assets
employed, working capital plus other
assets, and shareholders' equity. If
shareholders' equity (total assets - total
liabilities) is chosen, a portion of
long-term liabilities must be allocated
to the investment center to determine the
manager's resource base. One problem
with this definition of the resource base
is that, although it has the advantage of
emphasizing return to owners, it reflects
decisions at different levels of the
entity: short-term liabilities incurred by
the responsibility center (operating
decisions) and long-term liabilities
controlled at the corporate level
(long-term financing decisions).
Answer (C) is incorrect because
working capital plus other assets
reflects the assumption that the manager
controls short-term credit. However, no
corporate-level decision to allocate
long-term liabilities to subunits is
necessary.
Answer (D) is incorrect because they
reflect an assumption that the subunit
manager does not influence the resource
base (denominator of the ROI
calculation).
[92] Source: CMA 0692 3-14
Answer (A) is incorrect because a
production center may be a cost center,
a profit center, or even an investment
center. Transfer prices are not used in a
cost center. Transfer prices are used to
compute profitability, but a cost center
is responsible only for cost control.
Answer (B) is incorrect because an
investment center is not as fundamental
as a profit center.
Answer (C) is incorrect because
transfer prices are not used in a cost
center.
Answer (D) is correct. Transfer prices
are often used by profit centers and
investment centers. Profit centers are
the more fundamental of these two
centers because investment centers are
responsible not only for revenues and
costs but also for invested capital.

achieve its goals while functioning in


the best interest of the overall company.
Transfer prices can be determined in a
number of ways, including normal
market price, negotiated price, variable
costs, or full absorption costs. The
capacity of the Selling Division is often
a determinant of the ideal transfer price.
If the Fabricating Division had no
excess capacity, it would charge the
Assembling Division the regular market
price. However, if the Fabricating
Division has excess capacity of 1,000
units, negotiation is possible because
any transfer price greater than the
variable cost of $20 would absorb
some of its fixed costs and result in
increased divisional profits. Thus, any
price between $20 and $50 is
acceptable to the Fabricating Division.
Any price under $50 is acceptable to
the Assembling Division because that is
the price that would be paid to an
outside supplier.
Answer (B) is incorrect because the
Assembling Division would not pay
more than the market price of $50.
Answer (C) is incorrect because
Fabricating will not be willing to
accept less than its variable cost of $20.
Answer (D) is incorrect because
Fabricating should be willing to accept
any price between $20 and $50.
[94] Source: CMA 0696 3-26
Answer (A) is incorrect because
evaluating the seller is difficult if it can
pass along all costs to the buyer.
Answer (B) is incorrect because
transfers at full cost do not allow for a
seller's profit.
Answer (C) is correct. A transfer price
is the amount one segment of an
organization charges another segment
for a product. The selling division
should be allowed to recover its
incremental cost plus the opportunity
cost of the transfer. Hence, in a
competitive market, the seller should be
able to charge the market price. Using
full cost as a transfer price provides no
incentive to the seller to control
production costs.
Answer (D) is incorrect because a
full-cost transfer is favorable to the
buyer. It is lower than the market price.
[95] Source: CMA 0696 3-27
Answer (A) is incorrect because this
action is congruent with the goals of
Parkside. The use of idle capacity
enhances profits.

[93] Source: CMA 0694 3-30


Answer (A) is correct. An ideal
transfer price should permit each
division to operate independently and

Answer (B) is incorrect because the


transfer is at a loss (relative to full
cost) to the seller, although the company
as a whole will benefit.

Answer (C) is incorrect because the


buyer is indifferent as to whether to
purchase internally or externally.
Answer (D) is correct. If the seller has
excess capacity, it should lower its
transfer price to match the outside offer.
This decision optimizes the profits of
the company as a whole by allowing for
use of capacity that would otherwise be
idle.

determining the transfer price that meets


these criteria in all situations, a starting
point is to calculate the sum of the
additional outlay costs and the
opportunity cost to the supplier. Given
no idle capacity and a competitive
external market (all goods transferred
internally can be sold externally), the
sum of the outlay and opportunity costs
will be the market price.
[98] Source: CIA 1188 IV-23

[96] Source: CMA 0696 3-28


Answer (A) is incorrect because this
arrangement creates no disincentive for
the seller. It will make a profit on every
unit transferred.
Answer (B) is correct. Given that the
Plastics Division (the seller) has excess
capacity, transfers within the company
entail no opportunity cost. Accordingly,
the transfer at the negotiated price will
improve the performance measures of
the transferor. Purchasing internally at
below the market price also benefits the
transferee, so the motivational purpose
of transfer pricing is achieved. The goal
congruence purpose is also achieved
because the internal transaction benefits
the company.
Answer (C) is incorrect because the
market price charged by outside sources
is higher than the negotiated price.
Answer (D) is incorrect because, given
idle capacity, selling at any amount in
excess of variable cost should motivate
the seller.
[97] Source: CMA 1296 3-17
Answer (A) is incorrect because using
flexible budget cost as a transfer price
provides no motivation to the seller to
control costs and no reward for selling
internally when an external market
exists.
Answer (B) is incorrect because using
incremental cost as a transfer price
provides no motivation to the seller to
control costs and no reward for selling
internally when an external market
exists.
Answer (C) is incorrect because market
price is preferable to a budgeted or
actual cost with or without a markup
(unless the markup equals the profit
earned by selling externally).
Answer (D) is correct. Transfer prices
are the amounts charged by one segment
of an organization for goods and
services it provides to another segment
within the organization. Transfer prices
should promote congruence of subunit
goals with those of the organization,
subunit autonomy, and managerial
effort. Although no rule exists for

Answer (A) is incorrect because market


price is an approach to determine a
transfer price.
Answer (B) is correct. A transfer price
is the price charged by one segment of
an organization for a product or service
supplied to another segment of the same
organization.
Answer (C) is incorrect because outlay
price is an approach to determine a
transfer price.
Answer (D) is incorrect because
distress price is an approach to
determine a transfer price.
[99] Source: Publisher
Answer (A) is correct. Motivation is
the desire to attain a specific goal (goal
congruence) and the commitment to
accomplish the goal (managerial effort).
Managerial motivation is therefore a
combination of managerial effort and
goal congruence.
Answer (B) is incorrect because goal
congruence is the sharing of goals by
supervisors and subordinates.
Answer (C) is incorrect because
autonomy is the extent to which
individuals have the authority to make
decisions.
Answer (D) is incorrect because
managerial effort is the extent of the
attempt to accomplish a specific goal.
[100] Source: Publisher
Answer (A) is incorrect because the
retail price is the definition of the
market price, assuming an arm's-length
transaction.
Answer (B) is correct. At this price, the
supplying division is indifferent as to
whether it sells internally or externally.
Outlay cost plus opportunity cost
therefore represents a minimum
acceptable price for a seller. However,
no transfer price formula is appropriate
in all circumstances.
Answer (C) is incorrect because full
cost is the price usually set by an
absorption-costing calculation.

Answer (D) is incorrect because the


variable-cost-plus price is the price set
by charging for variable costs plus a
lump sum or an additional markup, but
less than full markup.

waste and maximize efficiency in a


competitive economy (an outside
market in which all padding produced
can be sold). This price also measures
the product's profitability and the
division managers' performance in a
competitive environment.

[101] Source: Publisher


Answer (A) is incorrect because the
price on the open market is the
definition of the market price.
Answer (B) is incorrect because outlay
cost plus opportunity cost is the price
representing the cash outflows of the
supplying division plus the contribution
to the supplying division from an
outside sale.
Answer (C) is incorrect because the
full-cost price is the price usually set by
an absorption-costing calculation.
Answer (D) is correct. The
variable-cost-plus price is the price set
by charging for variable cost plus either
a lump sum or an additional markup but
less than the full markup price. This
permits top management to enter the
decision process and dictate that a
division transfer at variable cost plus
some appropriate amount.

[103] Source: CIA 1190 IV-20


Answer (A) is incorrect because
inefficiencies are charged to the buying
department.
Answer (B) is correct. The optimal
transfer price of a selling division
should be set at a point that will have
the most desirable economic effect on
the firm as a whole while at the same
time continuing to motivate the
management of every division to
perform efficiently. Setting the transfer
price based on actual costs rather than
standard costs would give the selling
division little incentive to control costs.
Answer (C) is incorrect because, by
definition, cost-based transfer prices
are not adjusted by some markup.
Answer (D) is incorrect because
cost-based transfer prices provide the
advantages of clarity and administrative
convenience.

[102] Source: CIA 1191 IV-19


[104] Source: CIA 0595 III-96
Answer (A) is incorrect because the
market price will better achieve the
goals of a transfer pricing system. The
selling unit would not have as strong an
incentive to control costs if some
variant of actual cost is used. The
efficiency of the purchasing unit is also
promoted when it must treat the selling
unit as if it were an independent vendor.
Answer (B) is incorrect because the
market price will better achieve the
goals of a transfer pricing system. The
selling unit would not have as strong an
incentive to control costs if some
variant of actual cost is used.
Answer (C) is incorrect because the
market price will better achieve the
goals of a transfer pricing system. The
selling unit would not have as strong an
incentive to control costs if some
variant of actual cost is used.
Answer (D) is correct. The three basic
criteria that the transfer pricing system
in a decentralized company should
satisfy are to: (1) provide information
allowing central management to
evaluate divisions with respect to total
company profit and each division's
contribution to profit, (2) stimulate each
manager's efficiency without losing
each division's autonomy, and (3)
motivate each divisional manager to
achieve his/her own profit goal in a
manner contributing to the company's
success. The market price should be
used as the transfer price to avoid

Answer (A) is incorrect because return


on assets cannot be computed for a cost
center. The manager is not responsible
for revenue (return) or the assets
available.
Answer (B) is incorrect because return
on investment cannot be computed for a
cost center. The manager is not
responsible for revenue (return) or the
assets available.
Answer (C) is incorrect because the
payback method is a means of
evaluating alternative investment
proposals.
Answer (D) is correct. A cost center is
a responsibility center that is
responsible for costs only. Of the
alternatives given, variance analysis is
the only one that can be used in a cost
center. Variance analysis involves
comparing actual costs with predicted
or standard costs.
[105] Source: CMA 0697 3-29
Answer (A) is incorrect because
34.78% results from subtracting
working capital from plant and
equipment in calculating the net
investment.
Answer (B) is incorrect because
22.54% fails to include average
working capital in the total for the net

investment.
Answer (C) is incorrect because
19.79% results from not subtracting
general and administrative expenses in
the calculation of before-tax profit.
Answer (D) is correct. An investment
center is responsible for revenues,
expenses, and invested capital. Given
average plant and equipment of $1,775
and average working capital of $625,
the net investment is $2,400. Before-tax
profit is $400 ($4,000 sales - $3,525
cost of goods sold - $75 general
expenses). If before-tax ROI equals
before-tax profit divided by net
investment, the answer is 16.67% ($400
$2,400).
[106] Source: CMA 0697 3-24
Answer (A) is incorrect because the
methods use the same asset base.
Answer (B) is correct. Residual income
is the excess of the amount of the ROI
over a targeted amount equal to an
imputed interest charge on invested
capital. The advantage of using residual
income rather than percentage ROI is
that the former emphasizes maximizing
a dollar amount instead of a percentage.
Managers of divisions with a high ROI
are encouraged to accept projects with
returns exceeding the cost of capital
even if those projects reduce the
department's ROI.
Answer (C) is incorrect because the
methods use the same asset base.
Answer (D) is incorrect because use of
the residual income method requires a
knowledge of the cost of capital; thus,
arguments about the implicit cost of
interest may escalate with use of the
residual income method.
[107] Source: Publisher
Answer (A) is correct. Residual income
is the excess of the amount of the ROI
over a targeted amount equal to an
imputed interest charge on invested
capital. If a manager has $19,000,000
of invested capital ($17,200,000 of
plant and equipment + $1,800,000 of
working capital), a 15% imputed
interest charge equals $2,850,000.
Adding $2,000,000 of residual income
to the imputed interest results in a target
profit of $4,850,000. This profit can be
achieved if costs are $25,150,000
($30,000,000 revenue - $4,850,000
profit).
Answer (B) is incorrect because
Scenario 2 requires maximum costs of
$26,220,000 to reach the target.
Answer (C) is incorrect because
Scenario 3 requires maximum costs of
$25,330,000 to reach the target.

Answer (D) is incorrect because


Scenario 4 requires maximum costs of
$25,600,000 to reach the target.
[108] Source: CMA 0691 3-26
Answer (A) is incorrect because
supervisory salaries are costs
controllable by an assembly line
manager.
Answer (B) is incorrect because
materials are costs controllable by an
assembly line manager.
Answer (C) is incorrect because
repairs and maintenance are costs
controllable by an assembly line
manager.
Answer (D) is correct. Responsibility
accounting stresses that managers
should be held responsible for only
those factors under their control. Costs
are classified as controllable and
noncontrollable to assign responsibility,
which implies that some revenues and
costs can be changed through effective
management. For example, depreciation
on equipment is ordinarily not
controllable by the manager of an
assembly line and should not appear on
his/her performance report.
[109] Source: Publisher
Answer (A) is incorrect because the
return on sales was 3.33%.
Answer (B) is correct. Residual income
is the excess of the actual ROI in
dollars over a targeted amount equal to
an imputed interest charge on invested
capital. The rate used is ordinarily the
weighted-average cost of capital. Some
entities measure managerial
performance in terms of the amount of
residual income rather than the
percentage ROI. Assuming the
investment base is defined as total
assets available, Charlie's targeted
amount is $30,000 ($500,000 total
assets x 6% cost of capital). Assuming
that operating income of $25,000 is the
ROI in dollars, residual income was
$(5,000). This result is consistent with
defining the numerator of the ROI
calculation (Income Investment) as
operating income. However, it might
also be defined as net profit after taxes
(net income). Moreover, the ROI
denominator may be defined variously,
e.g., total assets available, total assets
employed, working capital plus other
assets, or shareholders' equity.
Answer (C) is incorrect because 6% is
the cost of capital.
Answer (D) is incorrect because
$(20,000) assumes that ROI in dollars
is $25,000 (operating income) and that
the targeted amount is $45,000 (6% x
$750,000 of sales).

[110] Source: CMA Samp Q3-2


Answer (A) is correct. The CM equals
revenues minus all variable costs
expensed. Given no WIP and no
beginning finished goods, the CM was
$25,200 [($100 - $30 - $20 - $10 $12) x 900 units]. The variable costs of
producing the units not sold are
included in ending inventory, rather than
in the CM. The fixed costs are also
excluded from computation of the CM.
Answer (B) is incorrect because
$28,000 results from assuming the sale
of 1,000 units.
Answer (C) is incorrect because
$31,500 results from assuming a UCM
of $35. This computation includes fixed
unit selling costs of $5 but excludes the
$12 per unit variable selling costs.
Answer (D) is incorrect because
$35,000 results from assuming a UCM
of $35 and sales of 1,000 units.

Answer (D) is incorrect because


$1,680,000 is the after-tax operating
income and does not deduct the cost of
capital.
[112] Source: Publisher
Answer (A) is incorrect because 8% is
the pre-tax cost of debt.
Answer (B) is incorrect because 8.89%
is based on the book value of equity.
Answer (C) is correct. The WACC is
an after-tax rate determined using the
fair values of the sources of long-term
funds. Thus, the appropriate cost of debt
is 6% [.08 x (1.0 - .25 tax rate)]
because interest is tax deductible.
However, the given equity rate (10%)
is not adjusted because distributions to
shareholders are not deductible. The
fair value of long-term debt is given as
$5 million. The book value of equity
must be $13 million ($20 million of
assets - $7 million of liabilities), and
its fair value is $15 million ($13
million + $2 million). Accordingly, the
WACC is 9%:

[111] Source: Publisher


Answer (A) is incorrect because
$731,240 is the cost of capital.
Answer (B) is correct. This problem
involves several steps. EVA equals
after-tax operating income minus the
product of the after-tax WACC and an
investment base equal to total assets
minus current liabilities. After-tax
operating income and WACC are not
given in the problem and have to be
calculated. WACC is an after-tax rate
determined using the fair values of the
sources of long-term funds. Interest is
tax deductible, so the coupon rate on
long-term debt has to be adjusted as
such, so that the appropriate cost of
debt for WACC calculations is 4.2%
[0.06(1 - 0.3 tax rate)]. The given
equity rate of 8% is not altered because
distributions to shareholders are not
deductible. The fair value of long-term
debt is given as $2,200,000. The book
value of equity has to be total assets
minus total liabilities, or $7,900,000.
Therefore, its fair value is $900,000
more than its book value, or
$8,800,000. The WACC is then
calculated as follows:
[(.042)($2,200,000 FV of LT debt)]
+ [(0.08)($8,800,000 FV of equity)]
WACC = ??????????????????????????????????? =
7.24%
($2,200,000 + $8,800,000
After-tax operating income is operating
income multiplied by 1 minus the tax
rate, which equals $1,680,000. Thus,
EVA is equal to ($1,680,000) [(0.0724)(10,100,000)], or $948,760.
Answer (C) is incorrect because
income taxes must be deducted from
operating income to compute EVA.

(.06 x $5 million FV of LT debt) +


(10% x $15 million FV of equity)
WACC = ??????????????????????????????????
($5 million + $15 million)
$300,000 + $1.5 million
= ???????????????????????
$20 million
= .09
Answer (D) is incorrect because 10%
is the cost of equity.
[113] Source: Publisher
Answer (A) is correct. EVA equals
after-tax operating income minus the
product of the after-tax WACC and an
investment base equal to total assets
minus current liabilities. Thus, EVA is
$1,380,000 {[$4 million x (1.0 - .25)] [.09 after-tax WACC x ($20 million
total assets - $2 million current
liabilities)]}.
Answer (B) is incorrect because
$1,620,000 is the required return on the
investment base.
Answer (C) is incorrect because
$1,830,000 is based on the assumption
that $13 million is the investment base.
Answer (D) is incorrect because
$3,000,000 is the after-tax operating
income.
[114] Source: CMA 0697 3-30
Answer (A) is incorrect because this
level of cost would result in a residual
income greater than $2,000,000.
Answer (B) is incorrect because this
level of cost would result in a residual

income greater than $2,000,000.


Answer (C) is correct. Residual income
is the excess of the amount of the ROI
over a targeted amount equal to an
imputed interest charge on invested
capital. If a manager has $19,000,000
of invested capital ($17,200,000 of
plant and equipment + $1,800,000 of
working capital), a 15% imputed
interest charge equals $2,850,000.
Adding $2,000,000 of residual income
to the imputed interest results in a target
profit of $4,850,000. This profit can be
achieved if costs are $25,150,000
($30,000,000 revenue - $4,850,000
profit).
Answer (D) is incorrect because
$25,690,000 results from subtracting
working capital from plant and
equipment in determining invested
capital.
[115] Source: Publisher
Answer (A) is incorrect because a
manager should be motivated to expend
the effort required to achieve
organizational objectives.
Answer (B) is incorrect because a
manager should be motivated to expend
the effort required to achieve
organizational objectives.
Answer (C) is correct. A managerial
control system should encourage the
efficient achievement of organizational
objectives. Thus, goal congruence and
managerial effort are aspects of
motivation. All managers should be
motivated to expend the necessary effort
to reach common goals.
Answer (D) is incorrect because a
manager should be motivated to expend
the effort required to achieve
organizational objectives.
[116] Source: Publisher
Answer (A) is incorrect because a
cost-based transfer price may provide
no incentive to minimize cost.

Answer (D) is incorrect because the


accrual return on investment in the first
years of a capital project may be very
low even though it has a positive net
present value.
[117] Source: CIA 1195 II-36
Answer (A) is incorrect because
employees will become discouraged if
the goals cannot be met.
Answer (B) is incorrect because goals
should be quantitative and specific.
They should not be too abstract.
Answer (C) is incorrect because not all
employees are superior. Employees are
discouraged by unreachable goals.
Goals should be based on an
employee's skills and capacity for
improvement.
Answer (D) is correct. Effective goal
setting requires a sufficient knowledge
of employees' jobs to set specific,
objective, verifiable goals. Employees
must also understand how goal-oriented
performance will be measured.
[118] Source: Publisher
Answer (A) is incorrect because
authority is the power to direct and
exact performance from others. It
includes the right to prescribe the means
and methods by which work will be
done.
Answer (B) is incorrect because
responsibility is the obligation to
perform.
Answer (C) is incorrect because
accountability is the liability for failure
to meet the obligation.
Answer (D) is correct. Controllability
is the extent to which a manager can
influence activities and related
revenues, costs, or other items. In
principle, controllability is
proportionate to, but not coextensive
with, responsibility.
[119] Source: CMA 0693 3-21

Answer (B) is correct. If the measure of


the retail store manager's performance
does not consider opportunity costs, for
example, the cost of financing
inventory, (s)he may be tempted to
make suboptimal decisions about
inventory levels. Hence, ordering
excessive inventory may help to
minimize the costs for which the
manager is responsible while
increasing the costs for which (s)he is
not.
Answer (C) is incorrect because the
organization may be better served if a
manager maximizes an absolute dollar
amount rather than a rate of return.

Answer (A) is incorrect because, if the


costs are not allocated to individual
managers, they have no reason to object
to the allocation.
Answer (B) is correct. Control is the
process of making certain that plans are
achieving the desired objectives. A
budget is one of the most common
control devices. It is a plan for the
future; it is not a contract. To interpret a
budget or other plan to be as inflexible
as a contract may encourage a manager
to act in ways contrary to the company's
best interest in a misguided effort to
meet the criteria proposed.

Answer (C) is incorrect because


participatory budgeting obtains the
support of those involved and is likely
to foster desirable behavior.
Answer (D) is incorrect because
changing budget targets as conditions
change results in setting fairer
performance goals.
[120] Source: Publisher
Answer (A) is incorrect because
disadvantages of participatory
budgeting and standard setting include
the cost in terms of time and money.
Answer (B) is incorrect because the
quality of participation is affected by
the goals, values, beliefs, and
expectations of those involved.
Answer (C) is incorrect because an
advantage of participatory budgeting is
that it yields information known to
employees but not to management.
Answer (D) is correct. Participatory
budgeting (grass-roots budgeting) and
standard setting use input from
lower-level and middle-level
employees. Participation encourages
employees to have a sense of ownership
of the output of the process. The result
is an acceptance of, and commitment to,
the goals expressed in the budget.
[121] Source: CMA 0693 3-26
Answer (A) is incorrect because
decentralizing decision-making
authority leads to more effective
solutions to operational problems.
Solutions will come from the
individuals with the greatest knowledge
of the operations involved.
Answer (B) is incorrect because, when
segments are autonomous, other
segments are regarded as external
parties, e.g., as suppliers, customers, or
competitors.
Answer (C) is incorrect because
autonomous segments may have the
authority to compete in the same
markets.
Answer (D) is correct. Decentralization
is beneficial because it creates greater
responsiveness to the needs of local
customers, suppliers, and employees.
Managers at lower levels are more
knowledgeable about local markets and
the needs of customers, etc. A
decentralized organization is also more
likely to respond flexibly and quickly to
changing conditions, for example, by
expediting the introduction of new
products. Furthermore, greater authority
enhances managerial morale and
development. Disadvantages of
decentralization include duplication of
effort and lack of goal congruence.

[122] Source: Publisher


Answer (A) is incorrect because they
are characteristics that Theory X
managers attribute to their employees.
Answer (B) is incorrect because they
are characteristics that Theory X
managers attribute to their employees.
Answer (C) is incorrect because they
are characteristics that Theory X
managers attribute to their employees.
Answer (D) is correct. MBO managers
believe that employees are committed
to achieving objectives, working hard
to receive the rewards of achievement,
and striving for self-actualization. The
MBO view is that employees enjoy
work, need little supervision, seek
responsibility, and are imaginative
problem solvers.
[123] Source: Publisher
Answer (A) is incorrect because
multiple measures are preferable.
Answer (B) is incorrect because
objectives should be specific.
Answer (C) is correct. Budgets and
standards are often the result of a
formal management-by-objectives
program. MBO is a top-down process
because the organization's objectives
are successively restated into
objectives for each lower level.
However, it is also a bottom-up process
because of the participation of
subordinates.
Answer (D) is incorrect because a
specific time period is established.
[124] Source: Publisher
Answer (A) is incorrect because MBO
goals may be set in terms of quantitative
measures (such as sales dollars) or
qualitative ones (such as improved
service).
Answer (B) is incorrect because it
describes management by exception.
Answer (C) is correct. The hallmark of
MBO is the mutual setting of goals by
the superior and the subordinate as a
basis for performance evaluation.
Based on the Theory Y philosophy that
employees want to work hard if they
know what is expected, MBO requires
top management participation and
commitment to the program, integration
of the objectives for all subunits into a
compatible system directed toward
accomplishment of overall goals,
provision for regular reporting of
performance, and free and honest
communication between superior and
subordinates. Subordinates must make
careful assessments of their abilities

and their interests, and managers must


"coach" subordinates rather than dictate
their proper goals. Both sides must
maintain flexibility to accommodate
unforeseen changes, and the review and
analysis of results before setting the
next round of goals is a vital part of the
process.
Answer (D) is incorrect because goal
setting should be participative.
[125] Source: Publisher
Answer (A) is incorrect because, when
an employee believes that (s)he is in
control, (s)he feels more responsible
for achieving goals.
Answer (B) is correct. MBO is the
process by which a manager and his/her
subordinate(s) work together to
formulate the objectives and goals of
the subordinate(s). One of the
weaknesses of MBO is that emphasis on
quantitative factors may cause
employees to focus on ends rather than
means. Thus, MBO may jeopardize the
quality of the organization's output.
Answer (C) is incorrect because
employee participation in goal setting
helps clarify goals and motivate the
employee.
Answer (D) is incorrect because an
employee's trust in the team approach
helps make the work atmosphere more
positive.
[126] Source: Publisher
Answer (A) is incorrect because market
share is an external nonfinancial
measure.
Answer (B) is incorrect because
delivery performance is an external
nonfinancial measure.
Answer (C) is incorrect because
customer satisfaction is an external
nonfinancial measure.
Answer (D) is correct. Feedback
regarding managerial performance may
take the form of financial and
nonfinancial measures that may be
internally or externally generated.
Moreover, different measures have a
long-term or short-term emphasis.
Examples of internal nonfinancial
measures are product quality, new
product development time, and
manufacturing lead time (cycle time).
[127] Source: Publisher
Answer (A) is incorrect because
current cost is an attempt to remedy the
theoretical deficiencies of historical
cost by presenting more accurate
balance sheet values.

Answer (B) is incorrect because current


disposal price is an attempt to remedy
the theoretical deficiencies of historical
cost by presenting more accurate
balance sheet values.
Answer (C) is correct. Historical cost
creates comparability issues because
returns on significantly depreciated
assets may be higher than those on
newer assets that have been acquired
using inflated dollars. Thus, otherwise
similarly situated managers may report
different operating results. Moreover,
managers may be reluctant to replace
aging assets.
Answer (D) is incorrect because
present value is an attempt to remedy
the theoretical deficiencies of historical
cost by presenting more accurate
balance sheet values.
[128] Source: Publisher
Answer (A) is incorrect because
self-interest provides an incentive to
maximize the measures used in
performance evaluation.
Answer (B) is correct. Effective
management control requires
performance measurement and
feedback. This process affects
allocation of resources to
organizational subunits. It also affects
decisions about managers'
compensation, advancement, and future
assignments. Furthermore, evaluating
their performance serves to motivate
managers to optimize the measures in
the performance evaluation model.
However, that model may be
inconsistent with the organization's
model for managerial decision making.
Answer (C) is incorrect because a
manager evaluated on the basis of
annual ROI has an interest in
maximizing short-term net income, not
long-term NPV.
Answer (D) is incorrect because the
models should be synchronized so that
the goals of the organization and the
manager are congruent.
[129] Source: Publisher
Answer (A) is correct. A manager who
does not control an activity may
nevertheless be the individual who is
best informed about it. Thus, a
purchasing agent may be in the best
position to explain price variances even
though (s)he cannot control them.
Moreover, if a manager is accountable
solely for activities over which (s)he
has extensive influence, the manager
may develop too narrow a focus. For
example, the manager of a cost center
may make decisions based only on cost
efficiency and ignore the overall
effectiveness goals of the organization.
By extending the manager's

responsibility to profits as well as costs


without changing his/her level of
control, the organization may encourage
desirable behavior congruent with
overall goals, such as improved
coordination with marketing personnel.

Answer (D) is incorrect because the


benefits-received criterion is preferable
when a cause-effect relationship cannot
be feasibly identified.
[132] Source: Publisher

Answer (B) is incorrect because the


potential for favorable behavioral
change may justify assigning
responsibility without control.
Answer (C) is incorrect because if
responsibility exceeds the extent to
which a manager can influence an
activity, the result may be reduced
morale, a decline in managerial effort,
and poor performance.
Answer (D) is incorrect because if
responsibility exceeds the extent to
which a manager can influence an
activity, the result may be reduced
morale.
[130] Source: Publisher
Answer (A) is incorrect because direct
costs can be traced to a particular cost
object in an economically feasible
manner.
Answer (B) is incorrect because current
cost is an attribute used to measure
assets.
Answer (C) is incorrect because
controllable costs can be influenced by
a particular manager.
Answer (D) is correct. Common costs
are the cost of products, activities,
facilities, services, or operations
shared by two or more cost objects.
They are indirect costs because they
cannot be traced to a particular cost
object in an economically feasible
manner. Hence, they must be allocated.
[131] Source: Publisher
Answer (A) is correct. The difficulty
with common costs is that they are
indirect costs whose allocation may be
arbitrary. A direct cause-and-effect
relationship between a common cost
and the actions of the cost object to
which it is allocated is desirable. Such
a relationship promotes acceptance of
the allocation by managers who
perceive the fairness of the procedure,
but identification of cause and effect
may not be feasible.
Answer (B) is incorrect because
allocation using an ability-to-bear
criterion punishes successful managers
and rewards underachievers.
Answer (C) is incorrect because
fairness is an objective rather than a
criterion. Moreover, fairness may be
interpreted differently by different
managers.

Answer (A) is incorrect because an


arbitrary allocation may skew operating
results.
Answer (B) is incorrect because the
allocation may create resentment and
conflict.
Answer (C) is correct. The allocation
reminds managers that support costs
exist and that the managers would incur
these costs if their operations were
independent. The allocation also
reminds managers that profit center
earnings must cover some amount of
support costs.
Answer (D) is incorrect because
efficient use of central support services
should be encouraged.
[133] Source: Publisher
Answer (A) is incorrect because a
static (fixed) budget is based on one
level of production and is not useful for
other activity levels.
Answer (B) is incorrect because a
static (fixed) budget is based on one
level of production and is not useful for
other activity levels.
Answer (C) is incorrect because a
continuous budget is revised (extended)
on a regular basis. It may or may not be
a flexible budget.
Answer (D) is correct. A flexible
budget is actually a series of several
budgets prepared for many levels of
sales. At the end of the period,
management can compare actual costs
or performance with the appropriate
budgeted level in the flexible budget.
Thus, flexible budgeting facilitates
variance analysis because it allows for
the effects of varying the activity level
so that price, efficiency, and other
variances can be isolated.
[134] Source: Publisher
Answer (A) is incorrect because the
balanced scorecard approach uses
multiple measures.
Answer (B) is correct. The trend in
managerial performance evaluation is
the balanced scorecard approach.
Multiple measures of performance
permit a determination as to whether a
manager is achieving certain objectives
at the expense of others that may be
equally or more important. These
measures may be financial or
nonfinancial and usually include items

in four categories: profitability;


customer satisfaction; innovation; and
efficiency, quality, and time.
Answer (C) is incorrect because the
balanced scorecard approach uses
financial and nonfinancial measures.

are overriding, common goals of the


parties. The conflict can be resolved if
the individuals involved understand that
it is preventing them from achieving
more important, mutually held goals.
[138] Source: CIA 0594 II-9

Answer (D) is incorrect because the


balanced scorecard approach uses
financial and nonfinancial measures.
[135] Source: CIA 0594 III-50
Answer (A) is incorrect because teams
support team members.
Answer (B) is incorrect because teams
make better decisions than individuals.
Answer (C) is correct. In a culture that
strongly emphasizes individual identity
and competition, the preference tends to
be for a clear link between effort and
outcome. However, teams tend to
submerge individual identity and
responsibility and therefore to blur the
link between individual effort and its
results.
Answer (D) is incorrect because teams
control and discipline members.
[136] Source: CIA 1193 III-2
Answer (A) is incorrect because, in an
organizational orientation session,
employees receive formal information
about objectives, the organizational
chart, benefits, and procedures.
Answer (B) is correct. A manager's
knowledge and skills are broadened by
serving in a number of different
capacities. To a significant degree,
research shows that managers learn to
manage by managing.
Answer (C) is incorrect because role
playing is assuming different roles in
different situations according to the
expectations of the group.
Answer (D) is incorrect because liaison
committees facilitate understanding
between management and employees.
[137] Source: CIA 0594 III-81
Answer (A) is incorrect because
conflict triggers include ambiguous
jurisdictions (unclear job boundaries).
Answer (B) is incorrect because
conflict triggers include competition for
scarce resources.

Answer (A) is incorrect because paying


a bonus is a positive reinforcement.
Answer (B) is correct. Negative
reinforcement removes an unpleasant
condition when the desired behavior
occurs, whereas positive reinforcement
rewards the desired behavior. Thus,
attending class is reinforced by the
removal of something unpleasant, i.e.,
the receipt of a written warning.
Because a warning is given after every
other absence, the reinforcement is
intermittent, not continuous.
Answer (C) is incorrect because
assigning a mentor is a positive
reinforcement. The firm is attempting to
link each individual with a positive role
model.
Answer (D) is incorrect because
holding a lottery is an intermittent
positive reinforcement.
[139] Source: CIA 0594 III-89
Answer (A) is incorrect because stating
the undesirable behavior clarifies for
the employee the link between conduct
and consequences.
Answer (B) is correct. Effective
discipline requires immediate
corrective action to eliminate the
negative effects of the undesirable
employee conduct and to establish and
reinforce appropriate behavior. Delay
merely invites more serious
consequences. Moreover, the
punishment should be commensurate
with the offense, and the employee
should clearly perceive the relationship
between the punishment and the
behavior.
Answer (C) is incorrect because, in
U.S. legal culture, the accused has the
right to be heard in his/her defense.
Answer (D) is incorrect because
focusing on the offense rather than the
offender is less likely to engender fear
and resentment on the part of the
employee.
[140] Source: CIA 0594 III-83

Answer (C) is incorrect because


conflict triggers include status
differentials.

Answer (A) is incorrect because


management can change people's
abilities, attitudes, expectations, and
behavior.

Answer (D) is correct. An appeal to


superordinate goals is a means of
resolving conflict. Superordinate goals

Answer (B) is incorrect because


management can change lines of
authority and responsibility, the nature

of jobs, and control mechanisms.


Answer (C) is correct. The factors a
change agent may alter can be classified
into three categories: structure,
technology, and people. Organizational
change is most likely to be an
adaptation to change in the environment.
Answer (D) is incorrect because
management can change how work is
performed.
[141] Source: CIA 1194 II-26
Answer (A) is incorrect because
superordinate goals are shared goals
that can be achieved only through
cooperation.
Answer (B) is correct. Smoothing is a
conflict resolution technique in which
differences are deemphasized and
common interests of the parties are
emphasized. It has the disadvantage of
not solving the underlying problems that
created the conflict.
Answer (C) is incorrect because
problem solving involves identifying
and correcting the source of the conflict.
Answer (D) is incorrect because
compromise requires each party to give
up something.
[142] Source: CIA 0595 II-32
Answer (A) is correct. A company's
mission statement defines crucial
organizational values and establishes a
basis for strategic planning. This
company's profit emphasis is an
example of a goals attainment approach
to measuring performance. The
emphasis is on the results attained, not
the means used in pursuing the results.
Answer (B) is incorrect because the
goals emphasis approach is not best
suited for companies in the formation
stage. Companies in the formative
stages need to focus on obtaining scarce
resources and developing a strategy for
long-run survival.
Answer (C) is incorrect because the
company has not effectively
implemented the strategic constituencies
approach. Only one of the organization's
important constituencies is considered.
Answer (D) is incorrect because the
company has not effectively
implemented the strategic constituencies
approach. Only one of the organization's
important constituencies is considered.
[143] Source: CIA 0595 II-35
Answer (A) is incorrect because the
most available information may not be
the best and most useful.

Answer (B) is incorrect because the


limitations on rationality suggest that
individuals will seek limited
information that is readily available and
familiar to them, thereby reducing the
effectiveness of decision making.
Answer (C) is correct. Bounded
rationality is the concept that recognizes
the limitations on an individual's ability
to process information. Thus, presenting
information in simpler forms assists the
decision maker. However, the danger of
constructing models that attempt to
manage complexity by isolating the
essential features of a situation is that
important elements may be omitted.
Answer (D) is incorrect because time
limitations only serve to reduce the
information available to the decision
maker and lead to decisions that are
similar to previous decisions.
[144] Source: CIA 0595 II-34
Answer (A) is correct. Group decision
making is often characterized by greater
acceptance of risk because of the
dispersal of accountability. Individual
decision making tends to be more
conservative because accountability can
be specifically assigned.
Answer (B) is incorrect because a
group has greater resources of
knowledge and experience than an
individual.
Answer (C) is incorrect because a
group has a wider diversity of views
and should be able to offer a wider
range of solutions.
Answer (D) is incorrect because a
disadvantage of group decision making
is that it is more time consuming than
individual decision making.
[145] Source: CIA 1195 II-1
Answer (A) is incorrect because
responsibility for group decisions is
more diffuse than when individuals
make decisions.
Answer (B) is incorrect because group
decision making almost always takes
more time than individual decision
making, except when the need for
diverse views is so great that an
individual decision maker needs to
consult many people or perform
research.
Answer (C) is incorrect because group
members usually have diverse views,
but their common need to be accepted
and respected by the group often
restrains the full, open expression of
their views when they fear strong
disagreement.
Answer (D) is correct. Groups tend to
be more creative than individuals.

Diversity of member views,


experiences, and abilities usually
results in considering more solutions to
a problem.

cheaper, but it may disrupt the


workplace and cause increased errors.
Classroom lectures are appropriate
when the skill training is too complex to
be conducted on the job.

[146] Source: CIA 1195 II-15


[149] Source: CIA 1195 II-33
Answer (A) is incorrect because a
leniency bias would result in ratings
that were consistently high for all
employees.
Answer (B) is incorrect because
leading questions would tend to
produce similar ratings for different
individuals, assuming all subordinates
are rated using the same questions.
Answer (C) is correct. The halo effect
is a source of systematic bias. It causes
a judgment about one characteristic to
affect judgments about others. For
example, a perception about one trait of
a person may influence perceptions of
other qualities of that individual.
Answer (D) is incorrect because, if the
supervisor were trying to treat everyone
the same, the variability among the
individuals rated would be lower.
[147] Source: CIA 1195 II-16
Answer (A) is incorrect because
authoritative command is a conflict
resolution technique.
Answer (B) is correct. Competition is a
conflict-handling intention
characterized by considerable
assertiveness (the degree to which the
party seeks to achieve his/her goals)
and a low degree of cooperativeness
(the degree to which the party attempts
to satisfy the concerns of others).
Encouraging competition stimulates
conflict.
Answer (C) is incorrect because
altering the structural variables is a
conflict resolution technique.
Answer (D) is incorrect because
compromise is a conflict resolution
technique.
[148] Source: CIA 1195 II-18
Answer (A) is incorrect because
classroom instruction is usually less
expensive than on-the-job training.
Answer (B) is incorrect because more
material can be covered in less time
with the lecture method.
Answer (C) is incorrect because any
level of depth can be achieved in the
lecture format.
Answer (D) is correct. The principal
advantage of learning by experience is
that what is learned is remembered
much better. On-the-job training is also

Answer (A) is incorrect because a


quality circle is a small group of
subordinates and supervisors, usually
eight to 10 people.
Answer (B) is incorrect because each
member is responsible for the success
of the circle, and success depends on
the ability of members to analyze and
solve problems.
Answer (C) is incorrect because quality
circles are used by companies to
accomplish objectives. Participation is
part of each worker's job.
Answer (D) is correct. Use of quality
circles is a form of participative
management. A quality circle is a group
of up to 10 individuals (managers and
subordinates) who do similar work and
who volunteer to meet weekly to
discuss and solve work-related
problems. However, management
retains the right to make the final
decisions.
[150] Source: CIA 1195 II-36
Answer (A) is incorrect because
employees will become discouraged if
the goals cannot be met.
Answer (B) is incorrect because goals
should be quantitative and specific.
They should not be too abstract.
Answer (C) is incorrect because not all
employees are superior. Employees are
discouraged by establishing
unreachable goals. Goals should be
based on an employee's skills and
capacity for improvement.
Answer (D) is correct. Effective goal
setting requires a sufficient knowledge
of employees' jobs to set specific,
objective, verifiable goals. Employees
must also understand how goal-oriented
performance will be measured.
[151] Source: CIA 1194 II-28
Answer (A) is incorrect because
classroom training permits the
employee to learn to use the actual
job-related equipment in a training
setting.
Answer (B) is incorrect because videos
provide the specific, technical details
necessary to perform the job and can be
viewed until the employee is
comfortable with the material.
Answer (C) is incorrect because

apprenticeships place the employee


with an experienced worker who serves
as a model for the understudy to
observe.
Answer (D) is correct. According to
Robbins (Organizational Behavior, pg.
565), simulation exercises, such as case
analyses, role playing, experiential
exercises, and group interaction
sessions, best serve in developing
problem-solving and interpersonal
skills.

group possesses greater resources than


an individual.
Answer (D) is correct. The groupthink
phenomenon is undesirable. Groupthink
occurs when group members accept
what appears to be the group consensus
rather than giving their honest input. The
result may be decisions with which
some members of the group are not
happy.
[155] Source: CIA 0596 II-20

[152] Source: CIA 1196 II-38


Answer (A) is incorrect because values
are stable and enduring.
Answer (B) is incorrect because
questioning values may result in their
reinforcement.
Answer (C) is incorrect because values
are relatively fixed and change only
slowly.
Answer (D) is correct. Values are
specific to each individual and involve
moral and personal issues. They tend to
be learned in childhood from parents,
friends, and others. Values can be
modified throughout life but ordinarily
tend to stay the same.

Answer (A) is incorrect because an


experienced employee or mentor from
the same company might be a good
source of information, but someone
hired at the same time will not have
better information about how the
company operates.
Answer (B) is incorrect because
outsiders will not know details of how
the corporate culture operates.
Answer (C) is correct. The corporate
culture of an entity consists of the
complex of shared values and
acceptable behaviors unique to the
organization. Although some companies
formally create a corporate culture and
provide explicit orientation regarding
it, new employees in most companies
must learn what is appropriate by
observation and experience.

[153] Source: CIA 0596 II-33


Answer (A) is incorrect because
personal beliefs alone are not an
appropriate basis for managerial action.
Answer (B) is incorrect because
personal beliefs alone are not an
appropriate basis for managerial action.
Answer (C) is correct. The only
legitimate grounds on which the
supervisor may take action is the
employee's behavior. Personal beliefs,
such as those on religious and political
matters, cannot be the basis of
personnel actions. Discrimination on
the basis of personal beliefs could
expose the organization to legal action.
Answer (D) is incorrect because
personal beliefs alone are not an
appropriate basis for managerial action.
[154] Source: CIA 1196 II-37
Answer (A) is incorrect because, if
members of the group are responsible
for the decision making, their
participation in the implementation
process will increase the ease with
which the decisions are carried out.
Answer (B) is incorrect because group
decision making adds legitimacy to the
solution by following democratic
methods.
Answer (C) is incorrect because a

Answer (D) is incorrect because, aside


from using valuable work time, this
approach also obtains worthwhile
information only from managers who
truly understand the corporate culture.
Moreover, the information may be
conflicting.
[156] Source: CIA 0596 II-32
Answer (A) is incorrect because
defining the task decreases both
acceptance and understanding of the
assignment. The staff member should
participate in the decision and be able
to discuss and clarify the assignment.
Answer (B) is correct. The supervisor's
expectations should be clear. The staff
member should also be involved in
determining how to reach the desired
outcome, thereby increasing both
acceptance and understanding of the
assignment.
Answer (C) is incorrect because this
approach almost guarantees that the
staff member will feel that (s)he failed
to perform well.
Answer (D) is incorrect because the
significant risk is that the staff member
will not define an outcome and an
approach with which the manager
agrees.
[157] Source: CIA 1196 II-26

Answer (A) is incorrect because an


approach based on pure power is an
autocratic style of leadership, not a
supportive approach.
Answer (B) is incorrect because the
custodial model depends on material
rewards for the worker. This model is
predicated on the belief that a happy
worker is a productive worker.
Answer (C) is correct. Supportive
management techniques orient workers
toward performance rather than
obedience or happiness. The leader
should have positive feelings for his/her
employees and should attempt to
encourage participation and
involvement. This approach is effective
when used with employees who are
motivated to work, improve themselves
and their abilities, and accomplish
goals.
Answer (D) is incorrect because the
manager's beliefs are not sufficient. The
workers must also believe in the
system.
[158] Source: CIA 1196 II-27
Answer (A) is incorrect because the
participative approach assumes that
workers are positively motivated.
Answer (B) is incorrect because the
presence of dissatisfiers is not
consistent with the participative
approach.
Answer (C) is correct. For a
participative management approach to
succeed, the parties must have sufficient
time, the issues must be relevant to
employees' interests, employees must
have the abilities (training and
communication skills) to participate,
and the company culture should support
participation. Accordingly, a limitation
of the participative approach is that it is
unlikely that all employees are willing
to participate in decision making.
Answer (D) is incorrect because such
conflicts arise when the needs of
individuals are not integrated with the
needs of the organization.
[159] Source: CIA 1196 II-32
Answer (A) is incorrect because
coercive power is rooted in fear or
threat of punishment.

Answer (C) is incorrect because expert


power is possessed by those who can
dispense valued information.
Answer (D) is incorrect because
legitimate power stems from holding a
superior position.
[160] Source: CIA 1196 II-34
Answer (A) is incorrect because
suspending employees is punishment.
Answer (B) is correct. Positive
reinforcement is a behavior
modification technique that provides
rewards for certain responses. It
focuses on desirable rather than
undesirable behavior. The practice of
praising employees when the detected
error rate in their work stays below a
predefined level demonstrates positive
reinforcement.
Answer (C) is incorrect because
eliminating time budgets is extinction,
which is the elimination of
reinforcement that is maintaining a
behavior.
Answer (D) is incorrect because not
requiring employees to work overtime
is negative reinforcement, which is the
elimination of something unpleasant
when a desired behavior occurs.
[161] Source: CIA 0596 II-22
Answer (A) is incorrect because
optimizing can usually discover more
benefits to divide.
Answer (B) is correct. Optimizing or
problem solving entails addressing the
source of conflict and finding
alternative strategies that benefit all
parties. It promotes cooperative,
positive attitudes that transfer to other
organizational behaviors. Hence,
optimizing may be worth the
expenditure of more resources than
other strategies because it improves the
future relationship of the parties.
Answer (C) is incorrect because, to
optimize, people need to think outside
of established habits to find new
benefits to divide.
Answer (D) is incorrect because
optimizing takes more time and energy
than other conflict resolution strategies.
[162] Source: CIA 1196 II-19

Answer (B) is correct. An individual


has referent power when (s)he
possesses attributes with which others
identify or to which they are highly
attracted. Thus, referent power induces
others to comply with an individual's
wishes based on that individual's
charisma, not level of authority and
expertise.

Answer (A) is correct. Diffusion


temporarily leaves the conflict
unresolved. Smoothing (downplaying
differences and emphasizing common
interests) and compromise (requiring
each party to make concessions) are
diffusion approaches to conflict
management. The disadvantage is that
the underlying problems remain

unresolved while the less controversial


issues are being addressed first.
Answer (B) is incorrect because
differences are downplayed while using
a diffusion approach.
Answer (C) is incorrect because
diffusion addresses issues of conflict.
Answer (D) is incorrect because
directly addressing the conflict is a
confrontational approach.

should be the criteria used.


Answer (C) is incorrect because faculty
members have no control over cost,
capital requirements, or revenue
produced.
Answer (D) is incorrect because faculty
members have no control over cost,
capital requirements, or revenue
produced.
[166] Source: CIA 1196 II-30

[163] Source: CIA 1196 II-33


Answer (A) is correct. The conflict
management technique that involves
face-to-face meetings is problem
solving. Problem solving is a means of
confronting the conflict and removing
its causes. The emphasis is on facts and
solutions, not personalities and
assignment of blame.
Answer (B) is incorrect because
expansion of resources addresses
conflicts that arise from scarcity.
Answer (C) is incorrect because the
manager is not using formal authority.
Answer (D) is incorrect because the
manager is not using behavioral
techniques to change attitudes and
behavior.

Answer (A) is incorrect because


evaluating outcomes is desirable when
the means are less important than the
ends.
Answer (B) is incorrect because use of
multiple evaluators increases the
probability of obtaining an accurate
appraisal.
Answer (C) is incorrect because use of
multiple criteria allows for a more
complete evaluation of quality.
Answer (D) is correct. Traits constitute
one of the weakest sets of criteria for
evaluating people. Many traits have
little or no relation to performance.
Furthermore, the evaluation of a trait is
a matter of judgment, and different
evaluators may have different views of
such traits as reliability, attitude, and
intelligence.

[164] Source: CIA 0596 II-27


[167] Source: Publisher
Answer (A) is correct. Being
considered a marginal performer may
cause the supervisor to feel insecure, a
factor that may distort his/her
perception of the worker's performance.
That the supervisor becomes more
critical as the worker accomplishes
more suggests that the supervisor is
threatened by the worker's superior
performance.
Answer (B) is incorrect because the
facts do not suggest that the worker's
attitude or his behavior is the cause of
the misperception.
Answer (C) is incorrect because the
facts say nothing about the work
environment.
Answer (D) is incorrect because the
facts say nothing about job standards.
[165] Source: CIA 1196 II-28
Answer (A) is incorrect because faculty
members have no control over cost.
Answer (B) is correct. The
performance of professional employees,
such as a college faculty, should be
measured using criteria over which they
have some control. In the case of a
college faculty, the quantity and quality
of output and the timeliness of output

Answer (A) is incorrect because the


change in the level of production
relative to the prior period does not
necessarily result in a difference in the
net income reported under variable and
absorption costing.
Answer (B) is correct. When
production exceeds sales, net income
reported under variable costing is
lower than net income reported under
absorption costing. Under variable
costing, total fixed manufacturing costs
are expensed in the current period.
Under absorption costing, fixed factory
overhead is inventoried. Thus, when
production exceeds sales, absorption
costing defers some fixed factory
overhead to the following period.
Answer (C) is incorrect because, when
production equals sales, net income is
the same under both absorption and
variable costing.
Answer (D) is incorrect because, when
sales exceed production, net income is
lower under absorption costing. Some
fixed costs from the prior period are
included in beginning inventory.
[168] Source: Publisher
Answer (A) is incorrect because

income will be lower under variable


costing when production exceeds sales.
Answer (B) is incorrect because
income will be higher under variable
costing.
Answer (C) is incorrect because
finished goods inventory will be higher
under absorption costing.
Answer (D) is correct. Absorption
costing always results in a higher
finished goods inventory because it
inventories fixed manufacturing
overhead. Under variable costing,
inventory valuation is lower because
those costs are expensed as incurred.

Answer (B) is incorrect because ending


inventory is $4,000 under absorption
costing.
Answer (C) is correct. Under
absorption costing, the unit cost of
ending inventory is $40 [$10,000 raw
materials + $20,000 direct labor +
$20,000 variable factory overhead +
$30,000 fixed factory overhead)
2,000 units produced]. Given no
beginning inventory, ending inventory is
100 units (2,000 units produced - 1,900
units sold). Hence, ending inventory is
$4,000 (100 units x $40).
Answer (D) is incorrect because ending
inventory is $4,000 under absorption
costing.

[169] Source: Publisher


Answer (A) is correct. Under variable
costing, operating income always
moves in the same direction as sales
volume. The reason is that all costs
included in cost of sales are variable
and therefore are directly related to
sales volume. Furthermore, variations
in production levels and inventories do
not affect profits and losses because no
fixed costs are capitalized.

[172] Source: Publisher

Answer (B) is incorrect because sales


volume is directly related to profit
under variable costing.

Answer (A) is correct. The difference


in profits between the two costing
methods is due to the treatment of fixed
factory overhead. Under absorption
costing, $40 per unit in ending inventory
is recognized on the balance sheet,
whereas only $25 is recognized under
variable costing. Hence, the $15
difference multiplied by 100 units
remaining in ending inventory causes
absorption costing profits to exceed
variable costing profits by $1,500.

Answer (C) is incorrect because sales


volume is directly related to profit
under variable costing.

Answer (B) is incorrect because $4,000


is ending inventory under absorption
costing.

Answer (D) is incorrect because sales


volume is directly related to profit
under variable costing.

Answer (C) is incorrect because


$30,000 is the total fixed factory
overhead for the period.

[170] Source: Publisher


Answer (A) is incorrect because ending
inventory is $2,500 under variable
costing.

Answer (D) is incorrect because


absorption costing profits will equal
variable costing profits only when sales
equal production.
[173] Source: Publisher

Answer (B) is correct. Under variable


costing, the unit cost of ending inventory
is $25 [($10,000 raw materials +
$20,000 direct labor + $20,000
variable factory overhead) 2,000
units produced]. Given no beginning
inventory, ending inventory is 100 units
(2,000 units produced - 1,900 units
sold). Hence, ending inventory is
$2,500 (100 units x $25).
Answer (C) is incorrect because ending
inventory is $2,500 under variable
costing.
Answer (D) is incorrect because ending
inventory is $2,500 under variable
costing.
[171] Source: Publisher
Answer (A) is incorrect because ending
inventory is $4,000 under absorption
costing.

Answer (A) is incorrect because


variable costing profits will exceed
absorption costing profits when
inventory quantities decrease during a
period.
Answer (B) is correct. Inventory
quantity will increase when production
exceeds sales. In that case, profit is
higher under absorption costing than
variable costing because some fixed
factory overhead is inventoried.
Answer (C) is incorrect because the
two methods report equal profits when
inventory remains the same during the
period.
Answer (D) is incorrect because
variable costing reports a lower
inventory value than absorption costing.
[174] Source: Publisher

method used.
Answer (A) is incorrect because, under
variable costing, all product costs are
variable.
Answer (B) is correct. Proponents of
variable costing maintain that fixed
overhead costs are more closely
correlated to capacity to produce than
to the production of individual units.
Thus, allocation of fixed production
costs is arbitrary.
Answer (C) is incorrect because
production costs (e.g., direct materials,
direct labor, and variable factory
overhead) are deemed to be
controllable.
Answer (D) is incorrect because period
costs are best attributable to time
intervals but nevertheless are necessary
to production.

Answer (B) is incorrect because, over


the life of a company, total profits will
be the same regardless of the costing
method used.
Answer (C) is correct. Over the entire
life of a company, all costs will
eventually be expensed. Thus, long-term
total profit will be the same under
variable and absorption costing.
Answer (D) is incorrect because, over
the life of a company, total profits will
be the same regardless of the costing
method used.
[178] Source: Publisher
Answer (A) is incorrect because
variable manufacturing overhead costs
are product costs under variable
costing.

[175] Source: Publisher


Answer (A) is incorrect because
income is not always greater using the
variable costing method.
Answer (B) is correct. Under
absorption costing, managers can
manipulate income simply by changing
the amount of units produced. Preparing
statements under variable costing
allows upper-level management to
better evaluate the activities of
production managers because some
fixed costs are not hidden in inventory.
Answer (C) is incorrect because the
method of costing is not useful in
evaluating sales personnel. They do not
control production costs.
Answer (D) is incorrect because
variable costing is not permitted to be
used for income tax purposes.
[176] Source: Publisher
Answer (A) is incorrect because all
variable manufacturing costs are
product costs under variable costing.
Answer (B) is incorrect because all
variable manufacturing costs are
product costs under variable costing.
Answer (C) is incorrect because all
variable manufacturing costs are
product costs under variable costing.
Answer (D) is correct. Under variable
costing, variable costs of production
(direct materials, direct labor, and
variable manufacturing overhead) are
treated as inventoriable costs.
[177] Source: Publisher
Answer (A) is incorrect because, over
the life of a company, total profits will
be the same regardless of the costing

Answer (B) is correct. Under variable


costing, fixed manufacturing overhead
costs are expensed in the period that
they are incurred. However, all
variable production costs are
inventoriable under variable costing.
Answer (C) is incorrect because, under
variable costing, selling and
administrative costs and fixed
manufacturing overhead are period
costs.
Answer (D) is incorrect because
variable costing inventories direct
materials, direct labor, and variable
manufacturing overhead.
[179] Source: Publisher
Answer (A) is incorrect because the
effect on absorption costing profits
cannot be determined without more
data.
Answer (B) is correct. Under variable
costing, profit always moves in the
same direction as sales volume.
Answer (C) is incorrect because the
effect on absorption costing profits
cannot be determined without more
data.
Answer (D) is incorrect because
operating profits moves in the same
direction as sales volume under
variable costing.
[180] Source: Publisher
Answer (A) is incorrect because the
total amount of fixed costs incurred is
reported in the income statement. No
fixed costs are absorbed by inventory
and expensed in a period other than
when they are incurred.

Answer (B) is incorrect because profits


and sales move in the same direction
under variable costing.
Answer (C) is incorrect because the
total amount of fixed costs incurred is
reported in the income statement. No
fixed costs are absorbed by inventory
and expensed in a period other than
when they are incurred.
Answer (D) is correct. Variable costing
emphasizes total costs by reporting the
total amount of fixed costs separately in
the income statement.
[181] Source: Publisher
Answer (A) is correct. One of the
advantages of variable costing is the
emphasis on marginal profit. This
emphasis serves as a justification for
changes in production levels. It is also
helpful in evaluating the profitability of
segments of a business.
Answer (B) is incorrect because it is a
disadvantage of direct costing.
Answer (C) is incorrect because it is a
disadvantage of direct costing.
Answer (D) is incorrect because it is a
disadvantage of direct costing.
[182] Source: Publisher
Answer (A) is incorrect because the
contribution margin is the difference
between sales and total variable costs.
Answer (B) is correct. The contribution
margin is the difference between sales
and total variable costs. It indicates
how much sales contribute toward
recovering fixed costs and providing a
profit.
Answer (C) is incorrect because the
contribution margin is the difference
between sales and total variable costs.
Answer (D) is incorrect because the
contribution margin is the difference
between sales and total variable costs.
[183] Source: Publisher
Answer (A) is incorrect because the
total variable delivery expense will
increase as production increases.
Answer (B) is incorrect because the
variance is favorable.
Answer (C) is incorrect because the
total variable delivery expense will
increase as production increases.
Answer (D) is correct. The variable
delivery expense should total $166,400
given sales of 52,000 units ($160,000
50,000 units = $3.20 per unit). Thus, the

variance is $3,400 ($166,400 $163,000). It is favorable since the


actual cost is less than that budgeted.
[184] Source: Publisher
Answer (A) is incorrect because the
variance is unfavorable.
Answer (B) is incorrect because the
variance is unfavorable.
Answer (C) is incorrect because a
flexible budget fixed cost should not
increase when production increases.
Answer (D) is correct. The budgeted
depreciation expense should be
$100,000 at all levels of production.
Thus, the variance will be $3,000
unfavorable ($100,000 budgeted $103,000 actual).
[185] Source: CMA Samp Q3-12
Answer (A) is incorrect because the
sales manager does not have
operational control of manufacturing
processes.
Answer (B) is correct. The production
manager is most likely to have influence
over and information about
manufacturing quality. This manager
may not control all facets of the
process, for example, the quality of
machinery or materials, but (s)he has
the knowledge to explain variances
between actual and expected
performance. (S)he may also be
motivated to exert pressure on those
who do control success factors.
Answer (C) is incorrect because the
division president does not have
operational control of manufacturing
processes.
Answer (D) is incorrect because the
receiving and inspection manager does
not have operational control of
manufacturing processes.
[186] Source: Publisher
Answer (A) is incorrect because the
board would be consulted initially only
if the immediate superior is the chief
executive officer and that person is
involved in the ethical conflict.
Answer (B) is correct. The Standards
of Ethical Conduct for Practitioners of
Management Accounting and Financial
Management state that the financial
manager/management accountant should
first discuss an ethical problem with
his/her immediate superior. If the
superior is involved, the problem
should be taken initially to the next
higher managerial level.
Answer (C) is incorrect because unless
"legally prescribed, communication of

such problems to authorities or


individuals not employed or engaged by
the organization is not considered
appropriate."
Answer (D) is incorrect because
resignation is a last resort.
[187] Source: Publisher
Answer (A) is incorrect because
"practitioners of management
accounting and financial management
have an obligation to the public, their
profession, the organization they serve,
and themselves, to maintain the highest
standards of ethical conduct."

responsible behavior clearly has


immediate costs to the entity, for
example, the expenses incurred in
affirmative action programs, pollution
control, and improvements in worker
safety. When one firm incurs such costs
and its competitor does not, the other
may be able to sell its products or
services more cheaply and increase its
market share at the expense of the
socially responsible firm. The rebuttal
argument is that in the long run the
socially responsible company may
maximize profits by creating goodwill
and avoiding or anticipating
governmental regulation.
[190] Source: Publisher

Answer (B) is incorrect because the


audit committee would be consulted
first only if it were the next higher
managerial level.
Answer (C) is correct. To resolve an
ethical problem, the financial
manager/management accountant's first
step is usually to consult his/her
immediate superior. If that individual is
involved, the matter should be taken to
the next higher level of management.
Answer (D) is incorrect because if the
superior is involved, the next higher
managerial level should be consulted
first.
[188] Source: Publisher
Answer (A) is incorrect because a
perfectly competitive market was
envisioned by classical economics.
Answer (B) is incorrect because the
concept embraces the public or societal
interest.
Answer (C) is correct. The concept of
corporate social responsibility involves
more than serving the interests of the
organization and its shareholders.
Rather, it is an extension of
responsibility to embrace service to the
public interest in such matters as
environmental protection, employee
safety, civil rights, and community
involvement.
Answer (D) is incorrect because the
concept embraces the public or societal
interest.
[189] Source: Publisher
Answer (A) is incorrect because such
behavior may prevent governmental
action.
Answer (B) is incorrect because each is
an argument for such behavior.
Answer (C) is incorrect because each is
an argument for such behavior.
Answer (D) is correct. Socially

Answer (A) is incorrect because this


course of action would be appropriate
only for the chief executive officer or
for his/her immediate subordinate when
the CEO is involved in the conflict.
Answer (B) is incorrect because the
proper action would be to present the
matter to the next higher managerial
level.
Answer (C) is incorrect because such
action is inappropriate unless legally
prescribed.
Answer (D) is correct. In these
circumstances, the problem should be
discussed with the immediate superior
unless (s)he is involved. In that case,
initial presentation should be to the next
higher managerial level. If the problem
is not satisfactorily resolved after
initial presentation, the question should
be submitted to the next higher level.
[191] Source: Publisher
Answer (A) is incorrect because it
applies to external auditors. The IMA
Code of Ethics does not expressly use
such language.
Answer (B) is correct. The preamble to
the IMA Code of Ethics states,
"Practitioners of management
accounting and financial management
have an obligation to the public, their
profession, the organizations they serve,
and themselves, to maintain the highest
standards of ethical conduct. In
recognition of this obligation, the
Institute of Management Accountants
has promulgated the following
standards of ethical conduct for
practitioners of management accounting
and financial management. Adherence
to these standards, both domestically
and internationally, is integral to
achieving the Objectives of
Management Accounting. Practitioners
of management accounting and financial
management shall not commit acts
contrary to these standards nor shall
they condone the commission of such
acts by others within their
organizations."

Answer (C) is incorrect because it


applies to external auditors. The IMA
Code of Ethics does not expressly use
such language.
Answer (D) is incorrect because it
applies to external auditors. The IMA
Code of Ethics does not expressly use
such language.
[192] Source: Publisher
Answer (A) is incorrect because it
states an aspect of the competence
requirement.
Answer (B) is correct. According to the
IMA Code of Ethics, financial
managers/management accountants must
"avoid actual or apparent conflicts of
interest and advise all appropriate
parties of any potential conflict."
Answer (C) is incorrect because it
states an aspect of the confidentiality
requirement.
Answer (D) is incorrect because it
states an aspect of the competence
requirement.
[193] Source: Publisher
Answer (A) is incorrect because, in this
situation, the chief executive officer is
the next higher managerial level.
Answer (B) is incorrect because the
immediate superior has promised or
taken action toward satisfactory
resolution.
Answer (C) is incorrect because the
immediate superior has promised or
taken action toward satisfactory
resolution.
Answer (D) is correct. According to the
IMA Code of Ethics, the financial
manager/management accountant should
"discuss such problems with the
immediate superior except when it
appears that the superior is involved, in
which case the problem should be
presented initially to the next higher
managerial level. If satisfactory
resolution cannot be achieved when the
problem is initially presented, submit
the issues to the next higher managerial
level. If the immediate superior is the
chief executive officer, or equivalent,
the acceptable reviewing authority may
be a group such as the audit committee,
executive committee, board of
directors, board of trustees, or owners."
[194] Source: Publisher
Answer (A) is incorrect because the
IMA Code of Ethics states that "except
where legally prescribed,
communication of such [ethical conflict]
problems to authorities or individuals

not employed or engaged by the


organization is not considered
appropriate."
Answer (B) is correct. According to the
IMA Code of Ethics, financial
managers/management accountants are
responsible for observing the standard
of confidentiality. Thus, the financial
manager/management accountant should
"refrain from disclosing confidential
information acquired in the course of
his/her work except when authorized,
unless legally obligated to do so."
Answer (C) is incorrect because the
financial manager/management
accountant should "inform subordinates
as appropriate regarding the
confidentiality of information acquired
in the course of their work and monitor
their activities to assure the
maintenance of that confidentiality."
Answer (D) is incorrect because the
financial manager/management
accountant is required to "refrain from
using or appearing to use confidential
information acquired in the course of
his/her work for unethical or illegal
advantage either personally or through
third parties."

You might also like